Anda di halaman 1dari 57

Hem- ONC 1

q 48

A 55-year-old Caucasian female presents to the emergency department with complaints of severe back pain and difficulty walking. She describes the back pain at the midthoracic level, which wraps around her upper abdomen in a band-like fashion bilaterally. She also complains of weakness, numbness, and tingling in both her lower extremities. Her symptoms started approximately two weeks ago and are progressively getting worse. She denies any difficulty with bowel or bladder function. She has a past history of nonHodgkin's lymphoma, which was appropriately treated eight months ago with chemotherapy and radiation therapy. Her neurological examination reveals a motor strength of 3/5 in both lower extremities. There is hyperreflexia of the knee and ankle reflexes, and a positive Babinski's sign bilaterally. Which of the following is the most appropriate next step in the management of this patient? A. Give low-dose corticosteroids and consult radiation oncologist B. Give high-dose corticosteroids and order MRI of the spine C. Emergent surgical decompression with posterior laminectomy D. Get MRI of the spine and call the surgeon for emergent surgical decompression E. Call the radiation oncologist for therapy This subscription is licensed to user: roopika only User ID: 123489 Explanation: Epidural spinal cord compression (ESCC) is a common complication of cancers, and it is sometimes the initial presentation of cancer. It frequently causes pain and some degree of neurological dysfunction. A metastatic tumor from any primary site can cause ESCC. Some of the common cancers with a tendency to metastasize to the spinal column are: prostrate, breast and lung cancer, non-Hodgkin's lymphoma, and renal cell carcinoma. This patient presents with a classic thoracic radicular pain (wraps around the abdomen) and neurological symptoms of ESCC. It is a medical emergency, and the treatment should be instituted as soon as possible. High-dose corticosteroids (especially dexamethasone) should be administered immediately, and MRI of the spine should be obtained to confirm the diagnosis. Corticosteroids exert their effect by decreasing the edema and swelling around the tumor tissue, thereby decreasing the compression. Radiation therapy is the definitive treatment of choice for most patients. The best response is seen in patients with radiosensitive tumors such as lymphoma, multiple myeloma, breast, and prostrate cancer. Once the MRI confirms the diagnosis, radiation therapy should be started.

(Choices A and B) High-dose corticosteroids are considered a part of standard therapy for ESCC. These should always be used along with radiation therapy. (Choices C and D) Lymphoma is extremely radiosensitive and should be treated with radiation therapy. Moreover, in studies, posterior decompression with a laminectomy with or without radiotherapy has not been shown to be superior to radiotherapy alone. Radical surgical resection with tumor debulking has shown to improve outcomes if done early and should be considered in patients with ESCC, due to tumors other than lymphoma. Educational Objective: Symptomatic epidural spinal cord compression should be treated emergently with a combination of high-dose corticosteroids and radiation therapy. MRI is the investigative procedure of choice. 72% of people answered this question correctly; This subscription is licensed to user ID: 123489 only A 23-year-old African American male who presents with fatigue and progressive dyspnea is hospitalized. Physical examination shows pallor. There is no jaundice. The lungs are clear on auscultation. The abdominal examination reveals no abnormalities. His initial lab test results are as follows: CBCHb 7.5g/dL MCV 82 fl Platelet count 180,000/cmm Leukocyte count 7,500/cmm Packed red blood cell transfusion is started. He becomes agitated and complains of severe bilateral flank pain while undergoing blood transfusion. He has received several blood transfusions in the past, but all of them were uncomplicated. His temperature is 38.3C(101F), blood pressure is 80/60 mm Hg and pulse rate is 120/min. Bilateral wheezing is heard on lung auscultation. His urine is dark. Which of the following is the best next step in the management of this patient? B. C. D. E. A. Stop transfusion and hydrate with IV normal saline Stop transfusion and give IV methyl prednisolone Stop transfusion and give IV furosemide Stop transfusion and administer IV antibiotics Continue transfusion and give supplemental oxygen

This subscription is licensed to user: roopika only User ID: 123489 Explanation:

Acute hemolytic transfusion reaction is a medical emergency that is usually caused by ABO incompatibility. Patients who require chronic red cell transfusions (e.g., AfricanAmerican patients with sickle cell anemia) may form multiple antibodies to common Rh, Kell, or other blood group antigens. The presence of such antibodies may increase the time required for a transfusion service to supply serologically compatible red cells. If undetected, these red cell antibodies may cause hemolysis, but severe hemolytic reactions are uncommon. In this case, the transfusion should be stopped immediately, and vigorous hydration should be started to treat the hypotension and to prevent renal failure. Low dose dopamine infusion and osmotic diuresis may also be employed. (Choice B) IV corticosteroids are used to treat allergic reactions. (Choice D) IV antibiotics are used in patients with septic reaction. (Choice C) Loop diuretics are commonly administered to treat volume overload. (Choice E) Continuing the transfusion may cause further complications such as hemolysis. Educational Objective: Patients with acute hemolytic transfusion reaction should be managed by immediately stopping the transfusion and vigorously hydrating the patient with normal saline (not Ringer's or dextrose) to treat the hypotension and to prevent renal failure. 65% of people answered this question correctly; This subscription is licensed to user ID: 123489 only The following vignette applies to the next 2 items A 56-year-old Caucasian male presents to your office for a routine check-up. He has no present complaints. He has long standing hypertension, which is controlled with amlodipine. Last year, he was hospitalized due to right-sided pneumonia. Three months ago, he was diagnosed with acute bronchitis. He smokes one pack of cigarettes daily and consumes alcohol occasionally. He denies any recreational drug use. His blood pressure is 130/80 mmHg, pulse is 90/min, temperature is 36.7C (98F), and respirations are 18/min.Hemoglobin 20.2 g/L Erythrocyte count 6.8 mln/mm3 MCV 90 fl Reticulocytes 1.5% Platelets 300,000/mm3 Leukocyte count 5,000/mm3 Neutrophils 58% Eosinophils 2% Lymphocytes 33% Monocytes 7%

Item 1 of 2 Which of the following is the most likely diagnosis in this patient? B. C. D. E. A. Polycythemia vera Relative polycythemia Secondary polycythemia Drug-induced polycythemia Thalassemia trait

This subscription is licensed to user: roopika only User ID: 123489 Explanation: Differential diagnosis of polycythemia may be difficult and may require sophisticated laboratory evaluation, such as measurement of serum erythropoietin levels and blood volume estimation. At the same time, a thorough history, physical examination and basic labs provide important clues to the correct diagnosis. Patients with secondary polycythemia present with appropriately high serum erythropoietin levels. Their erythropoietin production increases to compensate for hypoxemia which may be due to COPD, right-to-left cardiac shunt or high altitude residence. (Choices A and B) Unlike polycythemia vera or relative polycythemia, there is no increase in white blood cell count or platelet count in secondary polycythemia. (Choice D) Although some drugs (i.e., androgens and anabolic steroids) may result in polycythemia, Ca antagonists are not known to cause this blood disorder. (Choice E) Thalassemia trait may increase the RBC count; however, hemoglobin levels and MCV are decreased with this disease. Educational Objective: Erythropoietin production in patients with secondary polycythemia increases to compensate for hypoxemia. Unlike polycythemia vera, there is usually no increase in white blood cell count or platelet count in secondary polycythemia. 59% of people answered this question correctly; This subscription is licensed to user ID: 123489 only Item 2 of 2 Which of the following is the best next step in the management of this patient? A. Oxygen saturation measurement

B. C. D. E.

Liver function tests and urinalysis Chest x-ray Serum erythropoietin level Blood volume measurement

This subscription is licensed to user: roopika only User ID: 123489 Explanation: Polycythemia secondary to pulmonary diseases is the most common cause of polycythemia. A thorough history and physical examination may suggest the correct diagnosis. Low oxygen saturation on pulse oximetry, a very simple test, confirms the diagnosis. Sometimes, patients will not present with reduced oxygen saturation at rest or during the daytime. This is why pulse oximetry is obtained after minimal exertion in all patients. If suggested by the history, sleep studies may be necessary to determine the presence of nocturnal desaturation. (Choice C) A chest x-ray may help identify a pulmonary disease, but it does not confirm arterial desaturation, which actually leads to polycythemia. (Choice D) Serum erythropoietin levels are commonly measured in patients with polycythemia because it is an important diagnostic tool; however, this is not the first test to run in these patients. (Choice B) Liver function tests and urinalysis are employed when tumor polycythemia is suspected. (Choice E) Blood volume measurement is helpful in demonstrating relative polycythemia. Educational Objective: Pulse oximetry should be obtained after minimal exertion in patients with suspected secondary polycythemia. Sleep studies may sometimes be necessary to determine the presence of nocturnal desaturation. 33% of people answered this question correctly; This subscription is licensed to user ID: 123489 only The following vignette applies to the next 2 items A 6-year-old African American boy presents to the emergency department complaining of abdominal pain. He has a history of sickle cell disease and experienced an episode of sickle cell crisis six months ago. His medical history is otherwise unremarkable. Physical examination shows a thin boy in apparent distress. He is clutching his abdomen, whimpering, and lying uncomfortably on the examination table. His vital signs include a temperature of 38C (100.4F), blood pressure of 110/70 mm Hg, pulse of 92/min, and

respirations of 24/min. His tympanic membranes are pearly gray and the light reflex is present. No exudates or lesions are visible in the oropharynx. Chest auscultation is unremarkable and heart sounds are normal. He does not allow you to palpate his abdomen. Item 1 of 2 What is the next best step in the management of this patient? B. C. D. E. A. Intravenous fluids Ibuprofen PO for pain relief Ketorolac IV for pain relief Urinalysis and 24-hour measurement of urinary output Peripheral blood smear to evaluate for sickled cells

This subscription is licensed to user: roopika only User ID: 123489 Explanation: The sickle cell crisis is an episode of acute pain that can range in severity from minimal to agonizing. It is the most common type of vasocclusive event in patients with sickle cell disease. Although most of these crises have no identifiable cause, events such as dehydration, cold, infection, stress, menses, alcohol consumption, and nocturnal hypoxemia are known triggers. Some studies have also shown hemoglobin >8.5 g/dL to be a major risk factor for the development of an acute painful episode. Various areas of the body can be affected, with the most common locations including the back, chest, extremities, and abdomen. Objective clinical signs such as fever or tachypnea are seen approximately half the time. The first step in management of patients (especially children) presenting with an acute painful episode is the administration of fluids orally or intravenously to ensure optimal hydration (Choice A). Ibuprofen (Choice B) is not typically administered to patients in sickle cell crisis, as it does not provide adequate pain relief. Intravenous morphine is the most commonly prescribed analgesic in sickle cell patients. Another option used (especially in patients stricken with bone pain) is the non-steroidal anti-inflammatory drug ketorolac (Choice C), which provides pain relief superior to meperidine. However, fluid resuscitation should be performed first. Urinalysis and the measurement of urinary output (Choice D) would be particularly helpful if urinary tract infection or dehydration were suspected in a patient with sickle cell crisis. However, ordering these tests would not be the very first step in management of sickle cell crisis. In a patient with sickle cell disease, the peripheral blood smear (Choice E) usually reveals irreversible sickling of 5-50% of red blood cells. However, the presence of

sickled cells is not diagnostic of an acute pain episode. Unfortunately, there are no laboratory tests that can be used to differentiate this type of crisis from baseline condition. Educational Objective: The first step in the management of patients (especially children) presenting with sickle cell crisis is the intravenous administration of fluids. 68% of people answered this question correctly; This subscription is licensed to user ID: 123489 only Item 2 of 2 The first step in the management of this acute pain crisis is undertaken. The child now allows for a brief physical examination of his abdomen, which is significant for splenomegaly. What is the next best step in management of this patient? A. Blood and urine cultures B. Chest x-ray C. Complete blood count D. Intravenous antibiotics E. Warn the boy's parents that he should avoid contact sports until the splenomegaly resolves This subscription is licensed to user: roopika only User ID: 123489 Explanation: Young children with sickle cell disease have a 30% risk of developing splenic sequestration crisis (SCC) and are more susceptible because their spleens have not yet become fibrotic. In SCC, a dramatic fall in hemoglobin concentration occurs secondary to vaso-occlusion within the spleen and splenic pooling of red blood cells. Typically the spleen rapidly enlarges, and the patient can go on to experience hypovolemic shock. Patients with SCC who do not receive blood transfusions in time have a mortality rate of 10-15%. Therefore, a complete blood count (Choice C) should be ordered next for this child with splenomegaly. Blood and urine cultures (Choice A) are appropriate inclusions in the workup of febrile sickle cell patients and should ideally be obtained before antibiotics are administered. However, splenic sequestration crisis is of greater concern in this patient. Chest x-ray (Choice B) is an appropriate inclusion in the workup of febrile sickle cell patients, especially those with signs or symptoms of pulmonary involvement. However, splenic sequestration crisis is of greater concern in this patient.

Hospitalization and administration of intravenous antibiotics (Choice D) is recommended for those sickle cell disease patients in pain who are febrile (>40.0C), appear toxic, or are not receiving prophylactic penicillin. Patients with splenomegaly are at increased risk for splenic rupture, either from trauma or infiltrative disease that breaks the splenic capsule. Although the rupture itself may be painless, intraperitoneal bleeding, shock, and death can result. Therefore this child would be well advised to avoid contact sports until the splenomegaly resolves (Choice E). Since this warning is not immediately relevant, however, it can be delayed until discharge. Educational Objective: A complete blood count should be obtained immediately in sickle cell disease patients who present with abdominal pain and splenomegaly, as this patient group is at risk for developing splenic sequestration crisis. 54% of people answered this question correctly; This subscription is licensed to user ID: 123489 only The following vignette applies to the next 2 items A 36-year-old Caucasian female is seen in the emergency department (ED) for the evaluation of a sudden onset, left lower chest pain and difficulty in breathing. Approximately one year ago, she was diagnosed with left lower extremity deep vein thrombosis secondary to smoking and oral contraceptive pill (OCP) use. At that time, her OCPs were discontinued, she quit smoking, and she was treated with six months of warfarin therapy. She does not have any other medical problems. Her mother and maternal aunt have had "problems with blood clots." After performing the appropriate ancillary procedures, results reveal the presence of multiple, left-sided pulmonary emboli. A diagnosis of an inherited thrombophilia is suspected. Item 1 of 2 Which of the following is most likely to be present in this patient? B. C. D. E. A. Factor V Leiden mutation Low protein C levels Prothrombin gene mutation Antiphospholipid antibody Antithrombin III deficiency

This subscription is licensed to user: roopika only User ID: 123489 Explanation: The patient in the above scenario has suffered from a second episode of venous thromboembolism in a year. Factor V Leiden is the most common cause of inherited or

heredity thrombophilia, accounting for approximately 40-50% of the inherited thrombophilias. These disorders predispose the patient to the development of venous thromboembolism. (Choices B, C, D and E) Other common causes of heredity thrombophilia include prothrombin gene mutation, hyperhomocysteinemia, protein C, protein S, and antithrombin III deficiency. Some patients may have more than one type of inherited disorder, and are at even greater risk for venous thromboembolism. Educational Objective: Factor V Leiden is the most common cause of inherited or heredity thrombophilia, accounting for approximately 40-50% of the inherited thrombophilias. 65% of people answered this question correctly; This subscription is licensed to user ID: 123489 only Item 2 of 2 The results of the patient's hypercoagulation work-up are as follows:Factor V Leiden mutation Negative Protein C and S levels Normal Antithrombin III levels Normal Antiphospholipid antibody Negative Prothrombin gene mutation Positive MTHFR (methyltetrahydrofolate reductase) gene mutation Heterozygous Homocysteine levels Normal Which of the following is the most appropriate next step in the long-term management of this patient? B. year C. D. E. Start the patient on high dose folic acid (3 mg) per day Prescribe life long warfarin therapy Start her on folic acid 1 mg per day A. Prescribe warfarin for another one year Prescribe subcutaneous low-molecular-weight heparin for another one

This subscription is licensed to user: roopika only User ID: 123489 Explanation: The patient in the above scenario has suffered from a second episode of venous thromboembolism in a year. Her initial laboratory evaluation for an underlying inherited thrombophilia has revealed the presence of prothrombin gene mutation.

According to the current recommendations, all patients with a history of spontaneous venous thromboembolism in the presence of more than one inherited thrombophilia should have indefinite anticoagulation. Lifelong anticoagulation is also recommended for patients with hereditary thrombophilia who present with recurrent spontaneous thrombosis, life-threatening spontaneous venous thromboembolism (e.g. massive pulmonary emboli), or thrombosis at an unusual site (mesenteric or cerebral vein). (Choices A and B) All patients with recurrent thrombosis and evidence of a hereditary defect which predisposes them to venous thromboembolism should receive lifelong anticoagulation. (Choices C and E) Hyperhomocysteinemia is an independent risk factor for coronary, cerebrovascular, and venous thromboembolic disease. Therapy with daily folic acid supplementation has been shown to reduce the homocysteine levels in patients with hyperhomocysteinemia. The patient is heterozygous for MTHFR gene mutation and her homocysteine levels are normal; therefore, therapy with folic acid is not necessary. Educational Objective: All high-risk patients with a history of spontaneous thrombosis and a predisposing heredity defect should receive indefinite anticoagulation to reduce the risk of recurrent venous thromboembolism. 82% of people answered this question correctly; This subscription is licensed to user ID: 123489 only A 48-year old Caucasian man comes to the office complaining of asthenia, malaise and an unexplained weight loss. He does not use tobacco, alcohol or drugs. His family history is not significant. He takes no medications. His vital signs are within normal limits. Examination reveals mild splenomegaly. The laboratory tests show the following: CBCHb 11.4 g/dL Ht 35% MCV 84fl Leukocyte count 25,000/cmm Segmented neutrophils 65% Bands 8% Metamyelocytes 4% Blasts 5% Basophils 8% Lymphocytes 11% Monocytes 9% Additional tests are ordered after viewing the CBC results. These tests reveal low leukocyte alkaline phosphatase (LAP) activity, and the presence of the Philadelphia chromosome. Which of the following is the most effective strategy in this patient's management?

B. C. D. E.

A. Refer the patient to the hematologist to start hydroxyurea. Refer the patient to the hematologist to start busulfan. Refer the patient to the hematologist for bone marrow transplantation. Refer the patient to the hematologist to start Interferon alpha. Refer the patient to the hematologist to start cyclophosphamide.

This subscription is licensed to user: roopika only User ID: 123489 Explanation: Currently available treatment options for Chronic Myeloid Leukemia (CML) include: Bone marrow transplantation Oral chemotherapeutic agents, such as busulfan or hydroxyurea Parenteral Interferon alpha The newer oral tyrosine kinase inhibitors, such as imatinib (Gleevec) The introduction of tyrosine kinase inhibitors has made a significant impact in the treatment and prognosis of patients with CML; however, at this time, there are no clear guidelines on when to use these medications vs. when to use bone marrow transplantation. The above patient is a very good candidate for bone marrow transplantation (allogeneic hematopoietic cell transplantation, or HCT) and must be referred immediately. He is less than 50 years old, and has just been diagnosed with chronic myeloid leukemia in the chronic stable phase. Months later, he may progress to an accelerated phase, and in three to five years, he may develop a blastic crisis. Blastic crisis used to be refractory to treatment, however HCT may now be performed, and is effective in 5-15% of patients in the blastic phase. (Choice A) Hydroxyurea can control the disease temporarily, but HCT is far more effective in stopping the disease, and prevents the development of more advanced stages of the condition. (Choice B) Busulfan should not be used if the patient is a candidate for HCT, because its cumulative doses can lead to pulmonary toxicity. (Choice D) Interferon-alpha (IFN) or hydroxyurea are alternatives for early CML management; however, treatment with IFN will lead to cytogenetic (not molecular) remission, and a possibility of recurrence of the disease. (Choice E) Cyclophosphamide is only used in the preparation for HCT to prevent graft versus host disease (GVHD). Educational Objective:

Hematopoietic cell transplantation (HCT) is currently the most effective management for CML. The possibilities of success are higher in patients who: (1) are younger than 50 years, (2) have had the disease for less than a year, and (3) are in the stable chronic phase and have not taken busulfan. Patients who fulfill all these requirements must be referred immediately to the hematologist for HCT. 43% of people answered this question correctly; This subscription is licensed to user ID: 123489 only A 66-year-old African-American man is admitted to the hospital with complaints of constant low back pain for the last six weeks. The pain is insidious in onset, and is progressively getting worse. It does not radiate down his legs, and is not associated with any neurological symptoms. He has a history of hypertension, diabetes mellitus, and prostate cancer treated with radical prostatectomy six months ago. His medications include aspirin, amlodipine, and metformin. You suspect vertebral metastasis from the prostate cancer, and a radionuclide scan confirms your suspicion of the osteoblastic bony metastatic disease. Which of the following is the most appropriate initial treatment for this patient? B. C. D. E. A. Leuprolide alone Flutamide alone Diethylstilbestrol alone Chemotherapy with mitoxantrone Leuprolide and flutamide

This subscription is licensed to user: roopika only User ID: 123489 Explanation: This patient has presented with a metastatic prostate cancer with spread to the vertebral column. Metastatic prostate cancer remains a common clinical problem, despite early detection and treatment of the cancer. The most common site of spread is the bones, especially the axial skeleton. Skeletal metastases are usually osteoblastic, and should be suspected in a patient with prostate cancer presenting with a new onset or worsening of chronic back pain. A radionuclide bone scan is the single most useful imaging modality for assessing any new bony metastasis. Androgen depletion is the primary treatment for men with a metastatic prostate cancer. It is usually palliative rather than curative, and most patients lose the responsiveness to hormonal manipulations within two years. In most cases, luteinizing hormone releasing hormone (LHRH) agonists (leuprolide) are used as the first line therapy for advanced metastatic prostate cancer. These bind to the LHRH receptors in the anterior pituitary gland and cause an initial release of luteinizing hormone (LH) and follicle stimulating hormone (FSH), which causes a transient rise of testosterone levels. After approximately one week of continuous therapy, there is down regulation of LHRH receptors, causing a decrease in the levels of LH. This leads to a significant decrease in the levels of

testosterone. In the first week of therapy, the patient's symptoms tend to get worse during the initial testosterone surge. LHRH agonist monotherapy is contraindicated in patients with painful vertebral metastases and severe ureteral obstruction. These symptoms get worse during the initial testosterone surge seen in the first week of therapy. In such cases, an antiandrogen (e.g., flutamide) is given for one week before starting LHRH agonist monotherapy to block the effects of the initial testosterone surge. (Choice A) As mentioned above, LHRH agonist monotherapy is contraindicated in this patient. (Choice B) Antiandrogens used in patients with metastatic prostate cancer are flutamide, bicalutamide, and nilutamide. These block androgen receptors and prevent the growthpromoting effects of endogenous testosterone and dihydrotestosterone. These are associated with lower survival rates when used as a single agent. (Choice C) Diethylstilbestrol is a nonsteroidal estrogen, which exerts its effect by negative feedback action on the hypothalamic pituitary axis. It reduces the release of LHRH from the hypothalamus, thus decreasing the release of LH from the anterior pituitary. It is not available in the United States for commercial use, and its use is discouraged, due to the frequent side effects (e.g., myocardial infarction, stroke, and pulmonary embolism) associated with estrogen use. (Choice D) Patients with advanced, symptomatic, hormone-resistant prostate cancer can be treated palliatively with chemotherapy. The optimal regimen for the chemotherapy is uncertain. Educational Objective: LHRH agonists are the initial treatment of choice in patients with metastatic prostate cancer. The concurrent administration of an antiandrogen for a week prior to LHRH therapy reduces the initial symptom flare. 44% of people answered this question correctly; This subscription is licensed to user ID: 123489 only A 56-year-old Caucasian woman is diagnosed with metastatic breast cancer. Extensive work-up reveals several metastatic foci in the liver and right-sided pleural effusion. The tumor cells are estrogen- and progesterone-receptor negative. You start combined chemotherapy based on cyclophosphamide, but little response is observed after two weeks of therapy. After the appropriate work-up, you decide to add monoclonal antibodies to the treatment regimen and receive a good response. Which of the following is most likely correct about this patient's condition? B. C. A. The tumor is diploid The tumor has low S-fraction The tumor demonstrates p53 protein accumulation

D. E.

The tumor demonstrates HER-2/neu over-expression Serum level of cathepsin D is highly elevated

This subscription is licensed to user: roopika only User ID: 123489 Explanation: Over-expression of HER-2/neu (an oncogene that encodes a transmembrane receptor belonging to the epidermal growth factor receptor family) is traditionally associated with a worse prognosis in breast carcinoma. There is some clinical evidence that HER-2/neu over-expression may be associated with relative resistance to alkylating agents and probably endocrine therapy. Monoclonal antibodies against HER-2/neu (Trastuzumab, Herceptin) are effective in treating patients with over-expression of this oncogene. These also increase the patient's sensitivity to chemotherapeutic drugs. (Choices A and B) Tumor aneuploidy and high S-fraction are associated with a worse prognosis. (Choice C) Accumulation of p53 protein is also an unfavorable prognostic factor, but no therapy targeting this protein is available. (Choice E) The prognostic significance of cathepsin D is not clear. Elevated levels of cathepsin D may be associated with a worse prognosis. There is no available therapy that targets this lysosomal proteolytic enzyme. Educational Objective: Monoclonal antibodies to HER-2/neu oncogene can be added to the treatment regimen of patients with breast carcinoma who over-express this oncogene. These patients usually demonstrate a good therapeutic response to the antibodies and improved sensitivity to chemotherapeutic agents. 65% of people answered this question correctly; This subscription is licensed to user ID: 123489 only The following vignette applies to the next 3 items A previously healthy 62-year-old African-American woman is hospitalized because of shortness of breath and chest pain. She has had these symptoms for the past two days. She has no other medical problems, and takes no medications. She has smoked one pack of cigarettes daily for the last 20 years. Her temperature is 36.7 C (98 F), blood pressure is 130/80 mmHg, pulse is 98/min., and respirations are 22/min. The patient's pulse oximetry showed 94% on 4-liters of oxygen. V/Q scan of the chest reveals a high probability for pulmonary embolism. Her baseline labs reveal the following: CBCHb Hct 38% 13 g/dL

Platelet count 240,000/cmm Leukocyte count 8,000/cmm CoagsProthrombin time INR 1.06 14 sec

You start her on anticoagulation with unfractionated heparin and warfarin. Her symptoms gradually resolve over the next five days. On day six of her hospitalization, she complains of pain and pallor in her left arm. Physical examination reveals a pale and tender distal left arm with diminished pulses. The patient's labs reveal: CBCHb 12.6 g/dL Hct 37.2% Platelet count 40,000/cmm Leukocyte count 10,000/cmm Coagulation TestsProthrombin time 19 sec INR; 1.78 PTT 60 sec Item 1 of 3 Which of the following is the most likely cause of her condition? B. C. D. E. A. Subtherapeutic anticoagulation Heparin-induced thrombocytopenia Warfarin induced skin necrosis Heparin-induced skin necrosis Warfarin-induced thrombocytopenia

This subscription is licensed to user: roopika only User ID: 123489 Explanation: This patient had an arterial thrombosis of the left arm following the administration of unfractionated heparin. She has developed heparin-induced thrombocytopenia (HIT), which is a well-known complication of heparin therapy. Two forms of HIT have been recognized, depending upon the onset, clinical course, and severity of the disease. Type I HIT is seen within two days of initiation of heparin therapy, and is usually associated with a lesser degree of fall in the platelet count (nadir platelet count of

100,000/microliter). The platelet count usually returns to normal with discontinuation of heparin, and there are no clinical consequences. Type II HIT is a more serious immune-mediated disorder characterized by the formation of antibodies against heparin-platelet factor 4 complex. The heparin-platelet factor 4 antibody complex then binds to the platelet surface, causing platelet activation and aggregation, leading to thrombocytopenia and platelet-rich clots. It typically develops 4 to 10 days after the initiation of heparin therapy. Patients have the platelet count in the range of 30,000 to 60,000/microliter. Spontaneous bleeding is unusual. Immune-mediated HIT is associated with both venous and arterial thrombosis. The major manifestations of venous thrombosis are deep venous thrombosis, pulmonary embolism, venous limb gangrene, and cerebral sinus thrombosis. Arterial thrombosis can lead to strokes, myocardial infarction, and limb and organ (kidneys, mesenteric) ischemia. (Choice C) Warfarin-induced skin necrosis has been reported in patients within the first few days of taking high doses of warfarin. High-dose warfarin induces a transient hypercoagulable state by causing a rapid reduction in protein C levels on the first day of therapy. The lesions are seen over the extremities, trunk, and breasts. It is not associated with thrombocytopenia. (Choice D) Heparin-induced skin necrosis is also a well-known complication of unfractionated heparin use. It usually involves areas rich in fat, such as the abdomen, although distal extremities can also be involved. It presents as an area of erythema, which quickly progresses to purpura, hemorrhage, and necrosis. Most of these patients do not develop thrombocytopenia. (Choice A) Even though his PT/INR is subtherapeutic, his PTT is therapeutic. This means that the patient is well anticoagulated. Educational Objective: HIT should be suspected in patients receiving heparin anticoagulation if they present with thrombocytopenia, thrombosis with thrombocytopenia, or a > 50% fall in the platelet count, 4- 10 days after the initiation of treatment. 77% of people answered this question correctly; This subscription is licensed to user ID: 123489 only Item 2 of 3 Which of the following is the most appropriate next step in the management of this patient? A. Discontinue warfarin B. Discontinue unfractionated heparin C. Discontinue warfarin and unfractionated heparin D. Discontinue unfractionated heparin and warfarin; initiate low molecular weight heparin

E.

Discontinue heparin and warfarin; initiate argatroban

This subscription is licensed to user: roopika only User ID: 123489 Explanation: The first and the most important intervention in a patient with suspected or documented HIT is the immediate cessation of exposure to all heparin products. The patients with HIT still remain at risk for thrombosis, even after the discontinuation of heparin. All such patients should be anticoagulated with direct thrombin inhibitors such as lepirudin or argatroban. They should be used for the prophylaxis and treatment of patients with HIT with or without thrombosis. The choice of agent depends on the coexisting medical conditions. (Lepirudin should be used with caution in patients with renal insufficiency, while the dose of argatroban should be adjusted in patients with hepatic dysfunction.) (Choice B) Warfarin therapy alone may increase the risk of venous gangrene in patients with deep vein thrombosis. Its use should be avoided in the absence of other anticoagulants until the platelet count rises above 100,000/microliter. (Choice D) Low molecular weight heparin should not be substituted for unfractionated heparin because it can also cross react with heparin-induced antibodies, and induce more antibody formation. Educational Objective: Exposure to both heparin and warfarin should be discontinued in patients with HIT. They should be anticoagulated with direct thrombin inhibitors for the prevention or treatment of thromboembolic complications. 27% of people answered this question correctly; This subscription is licensed to user ID: 123489 only Item 3 of 3 Which of the following would have been the most useful strategy in preventing the above condition? B. C. D. E. F. A. Monitoring prothrombin time frequently Monitoring platelet counts frequently Substitution of unfractionated heparin with low molecular weight heparin Initiating warfarin after 3 - 5 days of heparin therapy Using higher doses of heparin early in the course of treatment Using lower doses of heparin early in the course of treatment

This subscription is licensed to user: roopika only User ID: 123489 Explanation:

The best way to prevent HIT is to use low molecular weight heparin (LMWH) instead of unfractionated heparin whenever possible. LMWH and heparinoids, such as danaparoid, are associated with a much lower incidence of HIT compared with unfractionated heparin. Another option is to limit heparin use to less than five days in order to prevent an antibody response, as well as the development of HIT. (Choice A) Prothrombin time is used to monitor the effects and adjust the dose of warfarin, not heparin. It does not affect the development of HIT. (Choice B) Monitoring the platelet counts will lead to early detection and treatment of HIT. It will not prevent the formation of heparin-platelet factor 4 antibodies and their clinical consequences. (Choice D) Initiating warfarin early can minimize the duration of heparin use, and prevent antibody formation. It should be started at the same time as (or within 24 hours of) heparin administration. (Choices E and F) Development of HIT is independent of the dose used, and has been reported to occur even with heparin flushes and the use of heparin-coated catheters. Educational Objective: The best way to prevent heparin-induced thrombocytopenia is by substituting unfractionated heparin with low molecular weight heparin. 40% of people answered this question correctly; This subscription is licensed to user ID: 123489 only The following vignette applies to the next 2 items A 45-year-old Caucasian female presents to your office with abdominal discomfort and distention that is especially bothersome after meals. She has noticed recently that she gets tired more easily. She feels "down", but denies any sleep difficulty, crying spells and suicidal ideation. She visited a doctor approximately one year ago due to similar complaints, and was told that the problem was psychological. She underwent myomectomy for a large uterine fibroid 8 years ago. She is not sexually active. Her family history is significant for pancreatic cancer in his father and a thyroid disease in her mother. The physical examination is significant for mild pallor of the conjunctivae. The abdomen is somewhat distended with no palpable masses. The laboratory values are:Hemoglobin 8.8 g/L MCV 117 fl Platelets 200,000/mm3 Leukocyte count 4,500/mm3 Neutrophils 56% Eosinophils 1% Lymphocytes 33% Monocytes 10%

Serum iron 110 mg/dL Serum cobalamin 90 pg/ml (N > 300 pg/ml) Serum folate 12.5 ng/ml (N > 4.0 ng/ml) Item 1 of 2 What is the best next step in the management of this patient? B. C. D. E. A. Schilling test Measurement of gastric secretion Auto-antibodies against intrinsic factor Auto-antibodies against vitamin B12 Bone marrow biopsy

This subscription is licensed to user: roopika only User ID: 123489 Explanation: This patient's findings (macrocytic anemia with a normal serum folate level and severely depressed serum cobalamin level) suggest the presence of B12-deficiency anemia. Accompanying gastrointestinal symptoms may indicate pernicious anemia as the cause of the problem. Pernicious anemia is characterized by the autoimmune destruction of parietal cells. The two major implicated auto-antibodies that have been described are the anti-parietal and anti-intrinsic factor (IF) antibodies. Anti-IF antibody testing is the recommended initial test for the detection of pernicious anemia. (Choice D) Anti-IF antibody testing has 50 to 84% sensitivity and almost 100% specificity in detecting pernicious anemia; anti-parietal testing is much less specific. (Choice A) Schilling test is a classic diagnostic test of pernicious anemia that uses radiolabeled cobalamin. It is more cumbersome than antibody testing, and can be used as the second-line test if the anti-IF test is negative. (Choice B) Patients with pernicious anemia are typically achlorhydric, but measurement of gastric secretions is generally not performed because it is highly invasive and nonspecific. (Choice E) Bone marrow biopsy can confirm the diagnosis of megaloblastic anemia, but is also highly invasive and non-specific for the underlying cause of the problem. Educational Objective: Anti-IF antibody testing has 50 to 84% sensitivity and almost 100% specificity in detecting pernicious anemia. It is the recommended initial diagnostic test. 47% of people answered this question correctly; This subscription is licensed to user ID: 123489 only

Item 2 of 2 After taking the appropriate initial steps, you proceed with gastric endoscopy. Which of the following findings is more likely to be present in this patient? B. C. D. E. A. Absent rugae in the fundus Prominent mucosal atrophy of the antrum Peptic ulcer Erosive gastritis Normal gastric mucosa

This subscription is licensed to user: roopika only User ID: 123489 Explanation: Pernicious anemia is associated with a type of gastritis called autoimmune metaplastic atrophic gastritis (AMAG). AMAG is caused by autoimmune aggression against gastric mucosa. An immune response is mainly directed against oxyntic cells and intrinsic factor. The three main components of AMAG are glandular atrophy, intestinal metaplasia and inflammation. Atrophy affects mainly the gastric body and fundus. (Choice B) Typically, little changes are observed in the gastric antrum. (Choices C and D) No erosions or ulcerations are usually observed. If present, these may indicate a high probability of malignancy. (Choice E) In advanced stages, the mucosa becomes thin and atrophic, and endoscopy may demonstrate absent gastric rugae in the body and fundus. Intestinal metaplasia replaces normal oxyntic glands; sometimes, the mucosa becomes villiform and resembles normal small intestine. Educational Objective: The three main components of AMAG (which is associated with pernicious anemia) are glandular atrophy, intestinal metaplasia and inflammation. Atrophy affects mainly the gastric body and fundus. Typically, little changes are observed in the gastric antrum. *High yield* 28% of people answered this question correctly; This subscription is licensed to user ID: 123489 only A 55-year-old Caucasian male is receiving doxorubicin-based chemotherapy for stage IV Hodgkin's lymphoma. He complains of an unstable gait and slowing of movements. Physical examination reveals a bilateral increase in muscular tonus without lateralizing neurologic signs. Which of the following is the best next step in the management of this patient's condition?

B. C. D. E.

A. Consider salvage chemotherapy. Switch to ondansetron from metoclopramide. Add a low-dose benzodiazepine. Decrease the dose of bleomycin. Decrease the dose of doxorubicin.

This subscription is licensed to user: roopika only User ID: 123489 Explanation: Metoclopramide is a prokinetic agent that acts as a central and peripheral D2-receptor blocker. It is used to treat (and prevent) chemotherapy-induced emesis; however, its use has been associated with extrapyramidal symptoms. The common central nervous system effects include akathisia, dystonia, and parkinsonian-like symptoms. When the drug is used in high doses, the incidence of these side effects may exceed 30%. Metoclopramide has now been largely replaced by 5 HT3 serotonin receptor antagonists (e.g. ondansetron). (Choice C) Low-dose benzodiazepine is used to reduce anticipation anxiety in patients undergoing chemotherapy. (Choices A, D, and E) Changing the chemotherapy regimen is not correct, because the symptoms are most probably related to the anti-emetic agent, not the chemotherapeutic drugs. Educational Objective: Metoclopramide is a central and peripheral D2-receptor blocker. Its use can result in drug-induced extrapyramidal symptoms such as akathisia, dystonia, and parkinsonianlike symptoms. 38% of people answered this question correctly; This subscription is licensed to user ID: 123489 only A 43-year-old male presents to your office with several months history of progressive fatigue and occasional palpitations. He had three episodes of acute respiratory infections over the last year; otherwise, his past medical history is insignificant. He smokes two packs of cigarettes daily and consumes alcohol occasionally. He is not taking any medications and denies drug abuse. He has been sexually active with several partners recently. The physical examination is insignificant. His laboratory values are:Hb 6.7 g/dL Red blood cells 2.3 million/cmm White blood cells 5,900/cmm Segmented neutrophils 60% Bands 3% Eosinophils 2% Lymphocytes 30%

Monocytes 5% MCHC 33% MCV 85 fL His HIV test results are negative. Which of the following is the best next step in the management of this patient? B. C. D. E. A. ESR and serum C-reactive protein Serum iron and TIBC Reticulocyte count Hemoglobin electrophoresis and Coombs' test Bone marrow biopsy

This subscription is licensed to user: roopika only User ID: 123489 Explanation: Normocytic/normochromic anemia is a very broad category. It may be caused by two large groups of disorders: 1) diseases with decreased red blood cell production, and 2) hemolytic disorders. To determine the predominant pathophysiologic mechanism of anemia in this patient, a reticulocyte count must be obtained. An elevated reticulocyte count indicates hemolysis as the cause of anemia; other findings associated with increased RBC destruction should be sought (e.g. elevated indirect bilirubin level, decreased haptoglobin, increased LDH, splenomegaly). A low reticulocyte count indicates a hypoproliferative state; renal disease, hypothyroidism and aplastic anemias may be present. (Choice D) Further evaluation of hemolytic anemias may include hemoglobin electrophoresis and Coombs' test. (Choice A) Anemia of chronic disease may present as normocytic/normochromic anemia; ESR and serum C-reactive protein measurements may help in the diagnosis. (Choice B) Iron deficiency anemia is microcytic/hypochromic. (Choice E) Bone marrow biopsy is not indicated at this stage. Educational Objective: Obtaining a reticulocyte count helps in determining the predominant pathophysiologic mechanism of normocytic/normochromic anemia. 48% of people answered this question correctly; This subscription is licensed to user ID: 123489 only A 40-year-old male is brought to the emergency room with confusion, lethargy, and decreased strength in his right arm. He is HIV-positive. He was diagnosed several months

ago, but he refused any kind of therapy. His most recent CD4 cell count was 40/L. A lumbar puncture is performed, but cerebral spinal fluid findings and cytology are inconclusive. A CT scan reveals a well-defined small focal lesion in the right cerebral hemisphere. Biopsy findings are consistent with primary CNS lymphoma. You decide to proceed with radiation therapy and corticosteroids, and to institute HAART. Which of the following is the best prognostic sign for this patient's condition? B. C. D. E. A. The stage of the lymphoma Decrease in EBV DNA in CSF Increase in soluble B-cell marker in CSF Increase in CD4 count Rapid response to radiation therapy

This subscription is licensed to user: roopika only User ID: 123489 Explanation: Primary CNS lymphoma (PCNSL) is a common malignancy in patients with advanced HIV infections, and is strongly related to the Epstein-Barr virus (EBV). It is usually associated with a significant degree of immunosuppression. Most patients with PCNSL have a persistently depressed CD4 count (< 50/L). Institution of HAART therapy is associated with an improved prognosis, especially in patients who demonstrate an improvement in the immune status (e.g. increase in CD4 count and decrease in viral load), because the degree of immunosuppression seems to be the major determinant of these patients' survival. (Choice E) Radiation therapy, combined with corticosteroids, induces a rapid response in 20-50% of patients; however, this does not seem to significantly prolong the overall survival. (Choice B) The measurement of EBV DNA levels in the CSF can be used to monitor the therapy; however, its prognostic role is not clear. (Choice C) An increase in the level of soluble B-cell marker in the CSF has been suggested as a diagnostic tool for PCNSL. (Choice A) PCNSL is almost always confined to the CNS. Staging is of value only in a minority of patients. Educational Objective: The degree of immunosuppression seems to be the major determinant of the survival in patients with PCNSL. 54% of people answered this question correctly; This subscription is licensed to user ID: 123489 only

You are the on-call resident for the oncology service in your hospital. A nurse calls you late in the evening and tells you that one of the patients from the oncology floor has started bleeding from his nose, and that the bleeding has not stopped with local pressure. The patient was diagnosed with acute myelogenous leukemia six months ago, and has been receiving chemotherapy ever since. He was readmitted earlier during the day with a relapse of AML. Later during the day, while eating dinner, he started bleeding from his left nostril. You order a stat complete blood count, which reveals a hemoglobin level of 8.2 g/dL and platelet count of 10,000/microL. You decide to transfuse the patient with six units of pooled platelets, and the repeat platelet count one hour after the transfusion is 15,000/microL. Which of the following is the most likely explanation for these findings? B. C. D. E. A. This is a normal response You have measured the platelet counts too early The patient has antibodies against the platelets You have not given him enough platelets He has disseminated intravascular coagulation

This subscription is licensed to user: roopika only User ID: 123489 Explanation: Platelets play a critical role in the normal hemostatic mechanism; hence, their deficiency or impaired function can cause significant or life-threatening hemorrhage. Platelet transfusions are commonly used to prevent or treat acute hemorrhage in patients with thrombocytopenia due to any cause. As a general rule, for an adult with a body surface area of 2.0 m2, a transfusion of 1 unit of platelets should immediately raise the post-transfusion platelet count by approximately 5,000/microL. (For this patient, six units should have raised the post-transfusion platelet count by at least 30,000/microL.) If there is a smaller than expected rise in the platelet count, refractoriness to platelet transfusions should be considered. Platelet refractoriness is defined as an absolute platelet count increment of less than or equal to 2,000/microL per unit of platelet transfusion given to an average-sized adult. Alloimmunization is one of the common and treatable causes of platelet refractoriness. It results from the production of antibodies to HLA class 1 antigen on the transfused platelets. These alloantibodies can cause a rapid destruction of the transfused platelets and can cause refractoriness to platelet transfusion. (Choice A) This is not a normal response. A normal response to platelet transfusion is an immediate post-transfusion rise in the platelet count of at least 5,000/microL per unit of platelets transfused. (Choice B) The post-transfusion platelet count should be measured 10 - 60 minutes after the completion of transfusion. A subnormal response or increment in the platelet count suggests platelet refractoriness due to alloimmunization.

(Choice E) A platelet count with a normal increment at one hour after transfusion, but with a rapid return to baseline within 24 hours indicates reduced platelet survival in the circulation. This is typically seen in patients with disseminated intravascular coagulation, sepsis, active bleeding, or in patients who use drugs that reduce platelet survival. (Choice D) The patient has received enough platelets. Six units of platelets should have raised the post-transfusion platelet count by at least 30,000/microL in this patient. Educational Objective: The post-transfusion platelet count should be measured 10 - 60 minutes after the completion of transfusion. Each unit of platelet transfused should cause an immediate post-transfusion rise in the platelet count by at least 5,000/microL. Platelet refractoriness due to alloimmunization should be considered if platelet transfusions fail to cause the expected increment in the platelet count. 35% of people answered this question correctly; This subscription is licensed to user ID: 123489 only A 46-year-old Caucasian female is hospitalized for upper gastrointestinal hemorrhage. She had two episodes of coffee ground colored vomiting during the past twelve hours. She has had intermittent episodes of passage of black stools over the last week. She visited a doctor two days ago and underwent an endoscopy that showed erosive gastritis and mild reflux esophagitis. Omeprazole therapy has been started. Her past medical history is significant for polycystic kidney disease and hypertension. Her most recent serum creatinine level is 2.9 mg/dL. Coagulation studies demonstrate the following results:APTT 30 sec (N 25-40 sec) PT 12 sec (N 11-15 sec) Bleeding time 15 min (N 2-7 min) Platelet count 250,000/mm3 Which of the following is the best next step in the management of this patient? B. C. D. E. A. Observation and supportive therapy IV desmopressin Platelet transfusion Cryoprecipitate infusion Immediate surgery

This subscription is licensed to user: roopika only User ID: 123489 Explanation: Renal dysfunction is associated with an increased tendency to bleed. Impaired platelet function is the major contributor to this tendency, and this is reflected by a prolonged

bleeding time. Other coagulation studies, such as prothrombin time (PT), partial thromboplastin time (PTT), and platelet count are usually normal. Correction of the platelet dysfunction is necessary in patients with active bleeding (such as the patient in this case) or those who are about to undergo a surgical procedure. Several available options to correct the abnormality include the following: correction of anemia, administration of desmopressin, dialysis, estrogen treatment, and cryoprecipitate infusion. Desmopressin represents the simplest and least toxic acute treatment of a prolonged bleeding time. It acts by increasing the release of factor VIII:von Willebrand factor multimers from endothelium. (Choice A) Asymptomatic patients do not need any specific therapy. (Choice D) Cryoprecipitate infusion is associated with infectious complications. Its use is limited to patients who are not responding to blood transfusions and desmopressin. (Choice C) Platelet transfusion is not indicated in such patients because their platelet count is usually normal. (Choice E) Any surgery without preceding correction of platelet dysfunction is very dangerous because it can precipitate significant bleeding. Educational Objective: Platelet dysfunction is the major contributor of increased bleeding tendency in patients with renal failure. Desmopressin represents the simplest and least toxic acute treatment of prolonged bleeding in patients with renal dysfunction. 38% of people answered this question correctly; This subscription is licensed to user ID: 123489 only A 74-year-old Chinese-American woman is brought to the emergency department. She has had fever, chills, productive cough, and shortness of breath for the past 24 hours. Her other medical problems include hypertension and rheumatoid arthritis. Her medications include enalapril and naproxen. Her temperature is 38.9 C (102 F), blood pressure is 150/90 mmHg, pulse is 106/min., and respirations are 22/min. Examination showed decreased breath sounds and crackles at the left base. Fecal occult blood per rectum was negative. Her laboratory tests reveal the following: CBCHb 9.4 g/dL MCV 84 fl Platelet count 450,000/cmm Leukocyte count 10,000/cmm Segmented neutrophils 84% Lymphocytes 16%

The chest x-ray showed an alveolar infiltrate in the left base. The patient was started on intravenous ceftriaxone and oral azithromycin. On the third day of treatment, additional hematologic tests are done, and these show the following:Ferritin 300 ng/mL (NV 15120) Iron 44 mcg/dL (NV 50-170) TIBC 200 mcg/dL (NV 250-450) Which of the following is the most appropriate diagnostic test for this patient's anemia? B. C. D. E. A. Erythropoietin levels Bone marrow biopsy Serum Transferrin Serum Transferrin saturation Erythrocyte sedimentation rate (ESR)

This subscription is licensed to user: roopika only User ID: 123489 Explanation: This patient has rheumatoid arthritis (RA), left lower lobe pneumonia, and normocytic, normochromic, anemia. Although RA usually causes anemia of chronic disease, a reliable diagnosis in this setting can only be established by performing a bone marrow biopsy. (Choices C and E) Because of the patient's acute infection, the ESR and ferritin levels will be abnormally increased, while the serum transferrin level and TIBC will be decreased. (Choice D) The serum transferrin saturation (serum iron/TIBC) and iron levels will not distinguish between anemia of chronic disease and iron deficiency anemia because both will present with diminished levels. (Choice A) Measurement of erythropoietin levels can help with decisions regarding the most adequate treatment of anemia of chronic disease in a patient with RA. This is not used to diagnose the anemia itself. Educational Objective: Anemia of chronic disease (ACD) can be difficult to distinguish from iron deficiency anemia in the setting of an acute bacterial infection. In this setting, the best test to confirm the diagnosis and rule out iron deficiency anemia is a bone marrow biopsy. 26% of people answered this question correctly; This subscription is licensed to user ID: 123489 only A 50-year-old African-American man comes to the emergency department (ED) because his left leg has been bothering him a lot. He has a vague and generalized pain in his left leg, and this is present both at rest and with activity. There is also marked swelling of the

left leg as compared to the right lower extremity. He was recently discharged from the hospital one month ago ago after being diagnosed and treated for deep venous thrombosis of the left leg. Since his discharge from the hospital, he has been taking 2.5 mg of warfarin daily in the morning. His INR (International Normalized Ratio) in the ED is 1.4. A venogram reveals recurrent acute venous thrombosis extending into the internal iliac vein and inferior vena cava. He does not have any other medical illnesses and is not taking any medications. At this point, which of the following would you recommend for this patient? A. Increase the dose of warfarin to bring INR within therapeutic range B. Switch him to Coumadin C. Interrupt the inferior vena cava with a filter D. Ask him to take the warfarin dose at bedtime E. Discontinue warfarin and begin thrombolytic therapy This subscription is licensed to user: roopika only User ID: 123489 Explanation: This patient has recurrent deep venous thrombosis with a more proximal extension into the inferior vena cava. This is most likely due to inadequate anticoagulation with warfarin. The anticoagulant effect of warfarin is mediated by the inhibition of vitamin K dependent gamma-carboxylation of factors II, VII, IX, and X. Although long-term therapy with warfarin is highly effective and causes a significant reduction in the frequency of recurrent venous thromboembolism, the dosage required to attain a therapeutic INR (International Normalized Ratio) is individualized for each patient. The dosage of warfarin is mainly determined by the difference in the genetically determined rate of drug metabolism and the patient's vitamin K status. Warfarin is metabolized in the liver by the hepatic cytochrome P450 2C9 isoenzyme, which exhibits multiple genetic polymorphisms that are responsible for the individualized dose requirements of patients. Furthermore, patients on a vitamin K-rich diet (green leafy vegetables) can have difficulty attaining a therapeutic INR; thus, they require higher doses of warfarin to achieve the desired INR. In this case, the patient's dosage of warfarin should be increased , and he should be followed closely with serial measurements of PT/INR. (Choice B) Warfarin is the generic form of Coumadin. Switching from warfarin to a different agent or brand name is not indicated at this point. (Choice C) Insertion of an IVC filter is usually indicated only in patients with acute venous thromboembolism who have a contraindication to anticoagulant therapy or who continue to have recurrent venous thromboembolism despite adequate anticoagulation.

(Choice D) Changing the dose schedule does not affect the bioavailability of the drug and is therefore not required. (Choice E) The use of thrombolytic therapy in the treatment of acute deep venous thrombosis is controversial and is associated with a higher complication rate. It is only considered if there is significant swelling which may compromise the arterial circulation to the extremity. Educational Objective: The dosage required to attain a therapeutic INR (International Normalized Ratio) is individualized for each patient. Insertion of an IVC filter is usually indicated only in patients with acute venous thromboembolism who have a contraindication to anticoagulant therapy or who continue to have recurrent venous thromboembolism despite adequate anticoagulation. 47% of people answered this question correctly; This subscription is licensed to user ID: 123489 only A 78-year-old African-American man comes to see you in the office for the evaluation of persistent, severe, low back pain. The pain does not radiate down to his legs, and is not associated with bladder or bowel disturbances. He has metastatic prostate cancer, which has progressed despite hormonal therapy and chemotherapy. A bone scan done six weeks ago revealed the presence of two vertebral metastases in the L4 and L5 vertebral bodies. He was started on high-dose narcotic analgesics at that time. He claims that the narcotics have "taken the edge" of the pain, but he continues to have severe debilitating pain. Neurological examination of the lower extremities is unremarkable. A repeat bone scan reveals results similar to the previous scan results. Which of the following is the next most appropriate step in controlling this patient's symptoms? B. C. D. A. Refer him for radiation therapy. Start him on high-dose hormone suppressive therapy. Admit the patient and start him on IV corticosteroids. Consult neurosurgery for surgical resection.

This subscription is licensed to user: roopika only User ID: 123489 Explanation: The axial skeleton is the most common site of spread of prostate cancer, which can cause significant pain and functional impairment. The primary goal of treatment in patients with metastatic, hormone-refractory prostate cancer is palliation. Specific aims in the management are adequate pain relief, improvement of the functional status, and prevention of vertebral compression fractures and subsequent spinal cord compression. Focal external beam radiation therapy is the treatment of choice for palliation of pain in patients with metastatic, advanced hormone-refractory prostate cancer. It should be used

in patients with a single or few focal bone lesions in which the pain is not adequately controlled with narcotic analgesics. (Choice B) The patient has a history of progressive, metastatic prostate cancer despite hormone therapy. It is unlikely that he will respond adequately to subsequent hormone manipulation. (Choice C) Corticosteroids reduce inflammation by inhibiting prostaglandin synthesis. These drugs are only useful when inflammation or edema surrounding the tumor causes pain secondary to compression or visceral distention. These are also useful in patients with epidural spinal cord compression. (Choice D) Surgery is only indicated in patients with pathological vertebral compression fractures or signs and symptoms of epidural spinal cord compression secondary to the metastases. Educational Objective: External beam radiation therapy should be used for pain alleviation in patients with single or few focal bone metastatic lesions due to hormone-refractory prostate cancer. 69% of people answered this question correctly; This subscription is licensed to user ID: 123489 only A 62-year-old Caucasian man comes to the office with complaints of a dull, nonthrobbing headache, hearing loss on the left side, and persistent tinnitus. His symptoms have been getting progressively worse for the last two weeks. He has a history of small cell cancer of the lung, which was treated with a combination of chemotherapy and radiotherapy four months ago. His lung mass has reduced considerably in size after its treatment. A contrast-enhanced magnetic resonance imaging scan shows a 1 x 2 cm circumscribed mass in the left cerebellopontine angle compressing the eighth cranial nerve. There is another small 0.5 x 1 cm mass in the left frontal lobe cortex with significant edema surrounding the lesion. Which of the following is the most likely cause of this patient's symptoms? B. C. D. E. A. Acoustic neuroma Medulloblastoma Metastatic cancer Meningioma Astrocytoma

This subscription is licensed to user: roopika only User ID: 123489 Explanation: Brain tumors are classified into tumors that originate in the brain itself (primary brain tumor), and those that originate elsewhere in the body and metastasize to the brain from another primary site (secondary brain tumor). Secondary brain tumors are the more

common type of brain tumors. The common primary sites of origin of brain metastasis, in the order of frequency, are: lung, breast, unknown primary, melanoma, and colon cancer. Small cell lung cancer has a predilection for early metastasis to the brain. The most common route of spread of the cancer is by hematogenous spread. The diagnostic imaging of choice to evaluate a patient with suspected metastatic disease is a contrastenhanced MRI scan. The clues to the presence of a metastatic brain lesion (as compared to a primary brain tumor) on radiographic imaging are the presence of multiple, wellcircumscribed lesions, and a relatively large amount of vasogenic edema as compared to the size of the lesion. In the above vignette, the patient's symptoms is most likely being caused by a metastatic lung cancer which has compressed the eighth cranial nerve. Educational Objective: Metastatic lesions to the brain are the most common cause of brain tumors. These account for more than half of all the brain tumors seen in adult patients. 67% of people answered this question correctly; This subscription is licensed to user ID: 123489 only A 78-year-old white male presents to your office because of a slowly growing, reddishbrown skin lesion on his forearm. He denies any recent trauma to his skin. He has no history of any contact allergies. He is a retired postal worker and spends most of his leisure time on a nearby beach. On examination, you find a 4 cm irregular, reddish-brown lesion near the elbow joint. The biopsy report reveals squamous cell carcinoma. Regional lymph node biopsy does not show any evidence of metastasis. He refuses to consent to any kind of surgical intervention. What is the most suitable alternative treatment for this patient? B. C. D. E. A. Chemotherapy with 5-Fluorouarcil Photodynamic therapy Interferon alpha-2b Radiation therapy Imiquimod

This subscription is licensed to user: roopika only User ID: 123489 Explanation: About 1.3 million cases of squamous cell cancer (SCC) and basal cell cancer are diagnosed every year in the United States. The treatment of choice for SCC is surgical excision. This has a 92% five-year cure rate for primary SCC. The best alternative treatment for patients who are not candidates for surgery is radiation therapy. The fiveyear cure rate for this procedure is approximately 90%. (Choice A) Chemotherapy with 5-FU may be an option for management of superficial SCC; however, this form of treatment produces more systemic effects compared to other available options.

(Choice B, C, E) All these new treatment options for SCC are still under investigation and are not currently recommended. Educational Objective: Surgery is the first line treatment for SCC. Radiation therapy is an alternative. 40% of people answered this question correctly; This subscription is licensed to user ID: 123489 only A 53-year-old Caucasian female is brought to the emergency department by her son due to a three-week history of headaches and behavioral changes. Her past medical history is significant for node-positive right breast cancer treated with a radical mastectomy and adjuvant chemotherapy six months ago. A CT scan shows multiple brain metastatic foci. Which of the following is the best statement concerning this patient's condition? B. C. D. E. A. Glucocorticoids are contraindicated Prophylactic anticonvulsive therapy is indicated Chemotherapy is the treatment of choice Whole brain radiation therapy improves survival Stereotactic surgery is indicated

This subscription is licensed to user: roopika only User ID: 123489 Explanation: The median survival of patients who present with brain metastatic disease is approximately one month if left untreated. Whole brain radiation therapy is the mainstay of treatment of multiple brain metastases. It can improve survival to three to six months. (Choice A) Corticosteroids help to increase patient survival by up to two months; therefore, these are generally recommended. (Choice C) Several chemotherapeutic regimens are being investigated for the treatment of brain metastases in patients with breast cancer; however, chemotherapy is currently not the treatment of choice due to the poor penetration of most chemotherapeutic drugs through the hemato-encephalic barrier. (Choice E) Stereotactic surgery may be indicated in patients with single brain metastasis and controlled systemic disease, but it is not usually recommended in multiple brain metastases. (Choice B) Prophylactic anticonvulsive therapy is not indicated in patients who do not present with seizures . Educational Objective: Whole brain radiation therapy is the mainstay of treatment of multiple brain metastases.

32% of people answered this question correctly; This subscription is licensed to user ID: 123489 only A 67-year-old Caucasian male presents to your office with a 3-week history of right shoulder pain. The pain is dull and poorly responsive to non-prescription analgesics. He denies any trauma to his shoulder. His past medical history is significant for prostate cancer that was treated with radical prostatectomy one year ago. He is concerned that the pain may be caused by the recurrence of the cancer. Physical examination is insignificant. No local tenderness or decrease in motion range is present. Which of the following is the best next step in the management of this patient? B. C. D. E. A. Chest x-ray Serum alkaline phosphatase Pelvic CT scan Radioisotope bone scan Reassurance and observation

This subscription is licensed to user: roopika only User ID: 123489 Explanation: Prostate cancer is the most common cause of metastatic bone disease in men. Radioisotope bone scan is a sensitive method of detecting bone metastasis and assessing the extent of the disease. It is recommended in patients who were previously treated for prostate cancer and who currently have skeletal complaints and/or rising PSA levels. (Choice A) Radioisotope bone scan is superior to radiography. (Choice B) Serum alkaline phosphatase gives no information about the localization of the process, but is helpful in monitoring a patient's response to treatment. (Choice C) Pelvic CT scan helps to detect local recurrences and pelvic pathology. (Choice E) Skeletal pain that is non-traumatic and unrelated to motion in patients with a history of cancer should raise the suspicion for metastatic bone disease. Educational Objective: Radioisotope bone scan is a sensitive method of detecting bone metastasis and assessing the extent of the disease. 53% of people answered this question correctly; This subscription is licensed to user ID: 123489 only A 59-year-old woman comes to your office because of progressive shortness of breath. She has no other medical problems, except borderline hypertension. She does not take any medications and denies cigarette or alcohol use. Her maternal aunt had breast cancer. Her annual breast exam, mammogram, and breast ultrasonogram, taken three months ago,

were all normal. Initial evaluation with chest x-ray shows pleural effusion without any obvious lung pathology. Echocardiogram reveals a normal ejection fraction, as well as normal systolic and diastolic functions of the heart. You perform a diagnostic pleurocentesis, and cytology reveals adenocarcinoma, which is ER (+) and PR (+). The repeat breast exam is completely normal. How do you manage this patient? B. tamoxifen C. D. E. A. Systemic treatment for metastatic breast cancer Bilateral modified radical mastectomy followed by chemotherapy and Bilateral random core biopsies of the breast Chemotherapy for metastatic ovarian cancer Radiation therapy

This subscription is licensed to user: roopika only User ID: 123489 Explanation: Breast cancers are notorious for being occult. Breast cancers can be missed, despite extensive workup. In this case, the histology findings and hormone status (ER and PR +) of the pleural fluid indicate metastatic breast cancer. There is no need to perform other tests (such as random core biopsies) to diagnose the actual primary tumor. The patient should be started on chemo and hormonal therapy for systemic metastasis. Chemotherapy is better than radiation therapy (Choice E) since it treats the entire body and may find stray tumor cells that migrated from the breast area. Hormonal therapy may be given because breast cancers, especially those that have ample estrogen or progesterone receptors, are frequently sensitive to hormonal changes. At present, tamoxifen is the most commonly prescribed hormone treatment. It is also used for breast cancer prevention. In the very elderly, tamoxifen alone has minimal side effects and may dramatically improve advanced breast cancer. Fulvestrant is another agent which has recently been approved in the United States. It is indicated for the treatment of hormone receptor-positive metastatic breast cancer in postmenopausal women with disease progression following anti-estrogen therapy (e.g., treatment with tamoxifen). Monoclonal antibodies are antibodies against proteins in or around a cancer cell. Antibodies recognize an "invader" (in this case, a cancer cell) and attack it. Monoclonal antibody therapy for breast cancer is currently being investigated. Trastuzumab (Herceptin) is an antibody against the HER-2 protein, a protein responsible for malignant cell growth in many women with breast cancer (about 30% of breast cancers). Educational Objective: The presence of estrogen receptor and progesterone receptor positive adenocarcinoma in the pleural fluid (even in the absence of any breast abnormality on examination, mammogram, and ultrasound) confirms metastatic breast cancer, and systemic therapy should be started. 27% of people answered this question correctly; This subscription is licensed to user ID: 123489 only

A 42-year-old Caucasian woman comes to the emergency department because of rightsided neck pain, persistent cough, and dyspnea for the past three days. Her past medical problems include hypertension and obesity. She takes enalapril and hydrochlorothiazide. She has smoked one pack of cigarettes daily for the past 25 years. She does not drink alcohol. She works as an executive secretary. Her temperature is 37.8C (100 F), blood pressure is 130/80 mm Hg, pulse is 92/min and respirations are 22/min. Examination shows decreased breath sounds in the upper part of the right lung. There is moderate swelling and erythema of the neck, moderate right jugular vein dilatation, and increased venous marks in the right arm. A chest x-ray reveals a right apical lung mass. Which of the following is the most appropriate immediate step in the management of this patient? B. C. D. results E. Order magnetic resonance imaging (MRI) of the neck and chest This subscription is licensed to user: roopika only User ID: 123489 Explanation: The patient's history, physical findings, and radiologic findings suggest superior vena cava syndrome (SVCS) secondary to a lung malignancy. At least 80% of the cases of SVCS are due to bronchogenic carcinoma. The classic presentation of SVCS begins with dyspnea, persistent cough, fascial fullness and neck pain, and progresses into hoarseness, dysphagia, chest pain and syncope. Pertinent physical findings are edema and erythema of the neck (which may sometimes compromise the face), and dilated veins of the arms and neck. Advanced disease is manifested by cyanosis, collateral veins in the thorax, ocular proptosis and lingual edema. The best diagnostic test for SVCS is a contrast CT scan of the chest and neck, which will reveal an obstruction of the superior vena cave due to the pulmonary mass, the metastatic nodes, or an intravenous thrombus. CT scan is very useful because it can reveal the extent of obstruction and provide a histopathologic diagnosis (via percutaneous biopsy), which will determine the particular therapeutic regimen required to manage the underlying malignancy. (Choice E) Magnetic resonance imaging (MRI) is a more expensive procedure, and does not offer any visual advantage over computed tomography. It is only used when a patient is allergic to the contrast dye used in CT scan, or when venous access cannot be obtained for contrast-enhanced studies. (Choice C) Immediate endotracheal intubation may be performed if there exists a risk of sudden airway occlusion, such as in patients with mediastinitis and deep neck infection; A. Start broad-spectrum antibiotic therapy Order a neck and chest computerized tomography (CT) scan with contrast Prepare for endotracheal intubation Order thyroid tests, CBC and chemistry panel, and reevaluate with the

however, this patient does not have a history of esophageal disease, upper respiratory tract infection, fever, or chills, to make us suspect mediastinitis. (Choice A) The erythema of the patient is due to venous occlusion and possible thrombosis, not to facial or neck cellulitis. Antibiotic therapy may be subsequently needed if the patient is found to have an infection; however, it is not necessary at this point. (Choice D) Biochemical and hematological tests may be obtained after CT scan evaluation. Educational Objective: Superior vena cava syndrome must be suspected in any patient who has a high risk of malignancy (e.g., family history or smokers) and presents with dyspnea, orthopnea, neck pain and swelling, and has cervical and upper extremities venous dilatation. The best diagnostic test to perform is computerized tomography of the neck and chest. 69% of people answered this question correctly; This subscription is licensed to user ID: 123489 only A 55-year-old Caucasian male presents to your office complaining of easy fatigability. He used to be physically active and always preferred to walk to work, which is 5 blocks away from where he lives. Now he says, 'Doc, I get short of breath when I walk even two blocks.' He denies any palpitations, chest pain, cough and lower extremity swelling. He does not feel down and has no problems with sleep. His past medical history is insignificant. He is currently not taking any medications. His blood pressure is 120/70 mmHg and heart rate is 95/min. The physical examination is significant for pallor of the conjunctivae and oral mucosa. His laboratory values are:Hemoglobin 7.2 g/L Erythrocyte count 3.5 mln/mm3 MCHC 27% MCV 72 fl Platelets 200,000/mm3 Leukocyte count 4,500/mm3 Neutrophils 56% Eosinophils 1% Lymphocytes 33% Monocytes 10% Serum ferritin 24 ng/mL Blood smear analysis shows microcytic erythrocytes with anisocytosis. What is the best next step in the management of this patient? B. C. D. A. Direct Coombs' test Hemoglobin electrophoresis Fecal occult blood test Upper GI endoscopy

E.

Serum B12 and folate levels

This subscription is licensed to user: roopika only User ID: 123489 Explanation: The diagnosis of iron deficiency anemia may not seem straightforward in this patient. Although microcytic/hypochromic anemia with anisocytosis is consistent with iron deficiency anemia, the serum ferritin value can raise doubts because the traditional ferritin cutoff point for iron deficiency is 15 ng/mL. Almost all patients with serum ferritin concentrations of less than 15 ng/mL are iron deficient. There is no other clinical situation that explains such an extremely low levels of serum ferritin. With the use of this cutoff point, the sensitivity of the test for detecting iron deficiency is only 59% and specificity is 99%. However, in one study, 25 % of women with absent stainable bone marrow iron had serum ferritin levels >15 ng/mL. Therefore, raising the cutoff point to 30 ng/mL increases the sensitivity to 92% and keeps specificity at 98%. Therefore, iron deficiency anemia is the most likely diagnosis in this patient. The GI tract is the major source of blood loss leading to iron deficiency anemia in males; therefore, a fecal occult blood test is a reasonable initial choice for this patient. (Choice D) If no historic clues are present (e.g., aspirin or NSAID use), colonoscopy is the best next step in the management of this patient. (Choice B) Although thalassemias can manifest as microcytic anemia, these usually present earlier in life, and characteristic findings on the blood smear may be observed ('target cells'). (Choice A) Autoimmune hemolytic anemia is usually normocytic/normochromic. (Choice E) Vitamin B12 and folate deficiency anemias are usually megaloblastic. Educational Objective: Almost all patients with serum ferritin concentrations of less than 15 ng/mL are iron deficient. There is no other clinical situation that explains such an extremely low levels of serum ferritin. The GI tract is the major source of blood loss leading to iron deficiency anemia in males. 86% of people answered this question correctly; This subscription is licensed to user ID: 123489 only A 46-year-old Caucasian female comes to see you in the office to follow-up her breast biopsy results. She does not have any family history of breast cancer. She had a screening mammography done about a month ago, which identified an area of microcalcifications that was suspicious for malignancy in her left breast. She subsequently underwent fine needle aspiration biopsy of the lesion. The biopsy results revealed the presence of lobular carcinoma in situ in the left breast. Which of the following is the most appropriate next step in the management of this patient?

B. C. D. E.

A. Regular examination and mammogram Wide surgical excision of the lesion Excisional biopsy with radiation therapy Perform an open biopsy of the lesion Wide surgical excision of the lesion with chemotherapy

This subscription is licensed to user: roopika only User ID: 123489 Explanation: Lobular carcinoma in situ (LCIS) is usually detected as an incidental finding on microscopic examinations of breast tissue. It arises from the lobules and terminal ducts of the breast. It is usually a multicentric (present at multiple sites in the breast) and bilateral disease. It is more commonly seen in younger premenopausal women, the incidence being much higher in white women, as compared to African-American women. Lobular carcinoma in situ is not considered to be the direct precursor of invasive breast cancer; however, its presence is one of the most important risk factors for the subsequent development of invasive breast cancer. The risk is increased in both the ipsilateral and contralateral breast, regardless of the side where LCIS is present. The various available options to manage patients with LCIS include close observation, tamoxifen therapy, and prophylactic bilateral mastectomy. Close observation with regular examinations and annual mammograms is appropriate for most women with LCIS. Studies have shown that there is no significant statistical difference in the mortality of women managed by close observation as compared to those managed by a surgical intervention or mastectomy. Patients can be followed regularly, just as any other patient with an increased risk of developing breast cancer. (Choice B) Surgical excision of the lesion with or without radiation therapy is generally not recommended because LCIS is usually a multicentric and bilateral disease. The only surgical modality that can be considered after extensive discussions with the patient is bilateral prophylactic mastectomy. (Choice D) Open breast biopsy of the lesion is usually not necessary. (Choice E) Chemotherapy has no present role in the management of patients with LCIS; however, tamoxifen has been shown to reduce the risk of development of invasive cancer and should be strongly considered in these patients. Educational Objective: The three possible options for the management of patient with LCIS include close observation, tamoxifen therapy, and bilateral prophylactic mastectomy. Surgical excision of the lesion, chemotherapy and radiation therapy have no role in the management of patients with LCIS.

10% of people answered this question correctly; This subscription is licensed to user ID: 123489 only A 55-year-old Caucasian woman comes to the physician because of weight loss, malaise, and weakness. Her other medical problems include COPD, diagnosed four years ago, and hypertension of ten years duration. She has smoked one-and-a-half packs of cigarettes daily for 28 years. She drinks 1-2 beers daily. Her medications include hydrochlorothiazide, metoprolol, albuterol metered dose inhalers, and ipratropium metered dose inhalers. Her physical examination reveals severe weakness in the proximal muscles and loss of deep tendon reflexes. Her vital signs are stable. Which of the following is the most appropriate next step in the management of this patient? B. C. D. E. A. Chest x-ray Serum CK and aldolase levels Acetylcholine receptor antibody Muscle biopsy Anti-Jo-1 antibody

This subscription is licensed to user: roopika only User ID: 123489 Explanation: Myasthenic syndrome or Lambert-Eaton syndrome is associated with small cell carcinoma of the lung. The patient's history of heavy smoking, weight loss, and muscle weakness makes her an ideal candidate to have this syndrome. Myasthenic syndrome results from autoantibodies directed against the voltage-gated calcium channels in the presynaptic motor nerve terminal. This leads to the defective release of acetylcholine and the manifestation of proximal muscle weakness. Paraneoplastic neurologic syndromes often precede the detection of the neoplasm; therefore, identifying the lung mass with a chest x-ray (or a CT scan of the chest, if CXR results are negative) is crucial. Electrophysiological studies confirm the diagnosis; the muscle response to motor nerve stimulation increases with repetitive stimulation. Treatment consists of plasmapheresis and immunosuppressive drug therapy. (Choice C) Autoantibodies against postsynaptic receptors explain the pathophysiology of myasthenia gravis. Reduction in the postsynaptic acetylcholine receptors leads to muscle weakness. The muscle weakness is provoked by repetitive or sustained use of the muscles involved, unlike myasthenic syndrome. Deep tendon reflexes are usually preserved; in fact, they may be somewhat brisk in clinically weak muscles. (Choices B, D and E) Immune-mediated muscle inflammation suggests polymyositis. It presents as proximal weakness characterized by difficulties in ascending and descending stairs, combing the hair, kneeling down, etc. It is not associated with absent reflexes. Measuring CK and aldolase levels, muscle specific antibodies, and muscle biopsy in this patient would therefore not be helpful .

Educational Objective: Eaton-Lambert syndrome is associated with small cell carcinoma of the lung, and results from autoantibodies directed against the voltage-gated calcium channels in the presynaptic motor nerve terminal. 48% of people answered this question correctly; This subscription is licensed to user ID: 123489 only The following vignette applies to the next 2 items A previously healthy, 30-year-old Caucasian female is admitted to the hospital. She has a rash over her lower legs and ankles. She first noticed the rash over her ankles 2 weeks ago, and it slowly spread up to her knees. She has no other past medical history. She denies any regular medication intake. Her vital signs are within normal limits. Physical examination reveals the presence of fine petechiae and purpura around her ankles and lower legs. The rest of her physical examination is normal. The labs reveal the following:Hb 14.4 g/dL MCV 90 fl Platelet count 16,000/cmm Leukocyte count 8,000/cmm Segmented neutrophils 60% Bands 3% Eosinophils 6% Lymphocytes 24% Monocytes 6% Prothrombin time 14 sec Partial thromboplastin time 30 sec Plasma fibrinogen 300 mg/dL D-dimer 0.22 g/mL Peripheral blood smear is normal. The admitting resident gets concerned with the lab results and orders 6 units of random donor platelet transfusion. The following day, the platelet count drops even further to 10,000/microliter. Item 1 of 2 Which of the following is the most likely cause of the drop in the platelet count? B. C. D. E. A. Disseminated intravascular coagulation Thrombotic thrombocytopenic purpura Antiplatelet antibodies Septicemia Drug induced thrombocytopenia

This subscription is licensed to user: roopika only User ID: 123489 Explanation: A presumptive diagnosis of idiopathic thrombocytopenic purpura (ITP) can be made when the history, physical examination, complete blood count, differential count, and peripheral blood smear do not suggest any causes for isolated thrombocytopenia. ITP occurs as a result of platelet destruction by specific autoantibodies. Platelet transfusions are rarely necessary to maintain the platelet count. The circulating platelets are rapidly removed by the autoantibodies anyway, making platelet transfusions futile. Patients with mild and asymptomatic thrombocytopenia (platelet count between 30,000 to 50,000/microliter) should not be treated. Transfusions are only necessary in severe or symptomatic thrombocytopenia (i.e. life threatening emergencies such as intracerebral or massive gastrointestinal hemorrhage). (Choice A, B, and C) There is no clinical or laboratory evidence of DIC, TTP or septicemia in this patient. (Choice E) The patient has not been exposed to any drugs which could cause or precipitate thrombocytopenia. Educational Objective: ITP is due to autoimmune phenomena involving the formation of antiplatelet antibodies. Treatment is only necessary in severe or symptomatic thrombocytopenia. 71% of people answered this question correctly; This subscription is licensed to user ID: 123489 only Item 2 of 2 Which of the following is the most appropriate next step in the management of this patient? B. C. D. E. A. Corticosteroids Plasmapheresis Low-dose heparin Intravenous immunoglobulins Give more platelets

This subscription is licensed to user: roopika only User ID: 123489 Explanation: ITP is acute and self-limiting in children, but it usually becomes a chronic disorder in adults. In such cases, corticosteroid therapy is the treatment of choice. Most adults respond to steroids within two weeks, with most patients responding within the first

week. IV IG can be used if steroids fail. A splenectomy is rarely performed in refractory cases. (Choice B) Plasmapheresis is the treatment of choice for hemolytic uremic syndrome (HUS) and thrombotic thrombocytopenic purpura (TTP). (Choice C) Heparin is used very cautiously in patients with early disseminated intravascular coagulation (DIC). Once the patient starts to bleed, heparin should not be used. Educational Objective: Corticosteroid therapy is the treatment of choice for ITP in adult patients. 43% of people answered this question correctly; This subscription is licensed to user ID: 123489 only A 48-year-old Caucasian woman comes to the physician for a follow-up visit. She was recently diagnosed of anemia. She currently complains of chronic and frequent tiredness, and daytime sleepiness. Her other medical problems include hypertension and rheumatoid arthritis. She takes diclofenac, enalapril and ferrous sulfate. She has no known drug allergies. Her social history is not significant. Her vital signs are within normal limits. Examination shows no abnormalities. Stool testing for occult blood is negative. Two months ago, when she was started on oral iron therapy, her baseline hemoglobin and hematocrit values were 8 g/dL and 26%, respectively. Her most recent laboratory results are as follows: CBCHb 8.2 g/dL Ht 27% MCV 88fl Platelet count 450,000/cmm Leukocyte count 6,000/cmm Segmented neutrophils 63% Lymphocytes 31% Monocytes 6% HematologyIron, serum 80 g/dL Iron binding capacity, serum 200 g/dL (normal: 250-370 g/dL) Ferritin 300 ng/mL Erythropoietin 1500 mU/mL (normal: 500-3600 mU/mL) ESR 40 mm/h Serum vitamin- B 12 level 440 pg/mL (normal: 200-800 pg/mL) Serum folic acid level 18 ng/mL (normal: 2.5-20 ng/mL)

Bone marrow studies reveal decreased numbers of sideroblasts and normal amounts of storage iron. Which of the following is the most appropriate intervention in the management of this patient? B. C. D. E. A. Continue ferrous sulfate tablets Parenteral iron Weekly erythropoietin Periodic blood transfusion Bi-weekly erythropoietin injections

This subscription is licensed to user: roopika only User ID: 123489 Explanation: Patients with chronic diseases may develop anemia (anemia of chronic disease, or ACD). Chronic conditions such as inflammation and malignancy gradually suppress red blood cell production in the bone marrow. Hematologic studies reveal a low serum iron, elevated ferritin levels, and usually, normal transferrin saturation. (Approximately 25% of ACD patients can have low transferrin saturation.) Around 20% of ACD patients have a hemoglobin concentration lower than 8 g/dL. In those cases, the cause of the anemia may be mixed; thalassemia, sideroblastic and iron deficiency anemia, as well as myelodysplasia have to be ruled out. This patient has a definite diagnosis of ACD made by bone marrow studies. Bone marrow studies reveal normal or increased amounts of storage iron in the macrophages, and a decreased number of sideroblasts. Treatment of the underlying disease and frequent follow-up visits are essential in the management of patients with ACD. Apart from treating the underlying cause, the 2 major options for treating anemia in these individuals are, erythropoietin injections or periodic blood transfusion. However, patients of ACD with erythropoietin levels more than 500 mU/mL usually do not respond to treatment with erythropoietin or darbepoetin. When the anemia is severe, such as in this patient, blood transfusions are necessary. (Choices A and B) Iron supplementation may be useful in patients with rheumatoid arthritis with co-existing ACD and iron-deficiency anemia, which can result from gastrointestinal bleeding secondary to drug therapy. Otherwise, iron therapy does not significantly help in the management of patients with ACD. Educational Objective: Treatment of the underlying disease (which is usually a chronic infection, autoimmune or inflammatory disease such as rheumatoid arthritis, lupus, and vasculitis) and frequent follow-up visits are essential in the management of patients with anemia of chronic disease. When the anemia is severe, blood transfusions are necessary. Treatment with erythropoietin can be successful if the serum levels of the hormone are under 500 mU/ml. If erythropoietin levels are normal and anemia is severe, periodic transfusions of packed red blood cells are indicated.

29% of people answered this question correctly; This subscription is licensed to user ID: 123489 only A woman calls you in the middle of the night to tell you that her husband has not been feeling well for the past two days. Her husband is a patient of yours, and he is a 76-yearold Caucasian man. He has not been eating well and feels very exhausted. He is barely able to stand up and walk on his own. According to his wife, she felt that he was a bit warm earlier in the day, and when she took his temperature at that time, it was elevated at 38.3C (101F). On direct questioning, she denies any recent history of rigors, chills, cough, shortness of breath, nausea, vomiting, abdominal pain, change in bowel habits, urinary symptoms, headache, visual symptoms, joint pains, or skin rashes. He has a past history of hypertension, atrial fibrillation, prostate hyperplasia, and osteoarthritis. He was recently diagnosed with acute leukemia and was discharged from the hospital ten days ago after a course of induction chemotherapy. His wife is concerned by his condition, and she is asking for your advice. Which of the following is the most appropriate response at this point? A. He most likely has viral infection. Please come to my office in the morning for further evaluation B. He most likely has bacterial infection somewhere. I can call to the local pharmacy for antibiotic prescription C. He most likely has anemia. Please come to my office in the morning for blood tests D. Please take him to the local emergency room immediately E. These are the side effects of his chemotherapy. Give him acetaminophen and come to my office tomorrow This subscription is licensed to user: roopika only User ID: 123489 Explanation: The patient in the above vignette is most likely suffering from neutropenic fever. Fever in a neutropenic patient is a medical emergency, and is defined as an isolated temperature reading of more than 38.3C or a continuous temperature of greater than 38C for more than one hour. Neutropenia is defined as an absolute neutrophil count of less than 1500 per microliter. Although the neutrophil count is not available at this point, it can be reasonably assumed that the patient has neutropenia. Cancer chemotherapy, especially induction chemotherapy for acute leukemia, often causes profound neutropenia, which can lead to potentially fatal infections, because majority of patients with neutropenia have occult bacterial infections. These patients do not exhibit the typical signs and symptoms of infection and usually have nonspecific symptoms of lethargy, malaise, and generalized weakness. All such patients should be instructed to immediately seek medical attention at the earliest signs of clinical deterioration. (Choices A,B,C and E) Any delay in intravenous antibiotic therapy in such patients can cause a significant increase in the mortality from these infections.

Educational Objective: Neutropenic fever in any patient should be considered a medical emergency. The patient can have a serious and life-threatening infection, even in the absence of typical signs and symptoms. 83% of people answered this question correctly; This subscription is licensed to user ID: 123489 only Mr. Clay is the brother of a nurse working at your floor. He is suffering from gallbladder cancer that has spread to the liver and all over his body. The nurse takes good care of him in her home, where he is living "the last days of his life in peace." He is currently complaining of a sharp, burning pain in his right thigh, which moves down to his right toes. The pain is 4/10 in intensity. He has tried taking ibuprofen for it, but this was not effective. The nurse wants you to give him morphine so that he can be peaceful. Which of the following should be given to the patient? B. C. morphine D. E. A. Extra strength NSAIDs Observe for two weeks as the pain will go away on its own A short-acting morphine for breakthrough pain followed by long-acting Desipramine Carbamazepine

This subscription is licensed to user: roopika only User ID: 123489 Explanation: This patient is having mild neuropathic cancer pain. Neuropathic cancer pains are divided in two types: 1. Sharp pain 2. Dull pain For sharp pain, the drug of choice is carbamazepine, and the alternate drug is valproic acid. Trials have been very successful in using gabapentin for such pain as well. (Choice D) For dull pain, the drug of choice is desipramine, and the alternate drug is imipramine. (Choices A and C) Neuropathic pain responds better to specific treatment than narcotics or NSAIDs. (Choice B) Observation in cancer pain is never acceptable. As the dictum goes, "never leave a dying patient in pain." Educational Objective:

In cancer patients with sharp neuropathic pain, the drug of choice is carbamazepine. For patients with dull neuropathic pain, the drug of choice is desipramine. 19% of people answered this question correctly; This subscription is licensed to user ID: 123489 only The following vignette applies to the next 3 items A 70-year-old woman comes to your office and complains of back pain, fatigue and inability to carry out her daily work. Her back pain began 18 months ago, and is slowly progressing in intensity. Her past medical history is remarkable for a hip fracture three months ago, when she slipped in her shower. She denies weakness in her extremities or paresthesias. She has mild constipation. Her urination is normal. She is currently not taking any medications. The initial lab tests show the following:Hemoglobin 12.0 g/L MCV 85 fl Platelets 220,000/mm3 Leukocyte count 4,500/mm3 Neutrophils 56% Eosinophils 1% Lymphocytes 33% Monocytes 10% ESR 15 mm/hr Serum creatinine 2.1 mg/dL Calcium 11.6 mg/dL Item 1 of 3 What is the best next step in the management of this patient? A. Vitamin B12 measurement, peripheral smear and bone marrow biopsy B. Skeletal survey, urinalysis and bone marrow biopsy C. Hemoglobin electrophoresis, urine electrophoresis and bone marrow biopsy D. Serum protein electrophoresis, urinalysis and bone marrow biopsy E. Serum protein electrophoresis, skeletal survey and bone marrow biopsy This subscription is licensed to user: roopika only User ID: 123489 Explanation: Multiple myeloma should be strongly suspected in elderly patients with bone pain (with or without pathological fractures), hypercalcemia, and renal dysfunction. Anemia and increase in the ESR are typical, but are not always present. The diagnostic work-up

includes: (1) serum protein electrophoresis; urine electrophoresis is often used to detect a monoclonal protein, (2) skeletal survey to assess the extent of skeletal involvement and identify any impending pathological fractures, (3) bone marrow biopsy to confirm the diagnosis by detecting sheets and clusters of plasma cells and calculating the percent of plasma cells in the aspirate. The other tests listed do not convey significant diagnostic information. (Choices B and D) Light chains are usually not detected by the urinary dipstick, which primarily senses albumin. (Choices A and C) Vitamin B12 measurement and hemoglobin electrophoresis are not useful in this case. Educational Objective: The diagnostic work-up in patients with suspected multiple myeloma includes serum protein electrophoresis, skeletal survey and bone marrow biopsy. 49% of people answered this question correctly; This subscription is licensed to user ID: 123489 only Item 2 of 3 You run all the appropriate tests on this patient. The bone marrow biopsy shows 20% plasma cells. While you are discussing the options for further investigation and treatment, the patient expresses her concern about future fractures and asks, "What can be done to reveal any potential skeletal lesions?" Which test can best address this patient's concerns? B. C. D. E. A. Whole body CT scan Whole body MRI Whole body X-ray Whole body PET scan Whole body Technetium-99m bone scan

This subscription is licensed to user: roopika only User ID: 123489 Explanation: A complete skeletal series (i.e., skull, long bones, and spine) should be performed once a patient is diagnosed with multiple myeloma. Skeletal surveys are important since these can assess the extent of skeletal involvement and sometimes even identify impending pathological fractures. Lytic lesions are characteristic for multiple myeloma. Conventional skeletal surveys reveal punched-out lytic lesions, diffuse osteopenia, or fractures in nearly 80 percent of patients at the time of diagnosis. (Choice E) Bone scans primarily detect blastic lesions (not lytic lesions). Furthermore, technetium-99m bone scanning is much less sensitive for lytic lesions compared to

conventional roentgenography, and is therefore not recommended. Multiple myeloma patients usually do not have elevated bony alkaline phosphatase levels. (Choices A and B) CT scan or MRI can be employed in patients with bone pain and no identifiable lesions on conventional X-ray. MRI is also often used to assess the integrity of the spinal cord in patients with spinal lesions. Educational Objective: A complete skeletal series (i.e., skull, long bones and spine) should be performed once a patient has been diagnosed with multiple myeloma. Bone scans are not useful for these patients. 27% of people answered this question correctly; This subscription is licensed to user ID: 123489 only Item 3 of 3 You inform the patient about the prognosis, as well as risks and benefits of therapy. She decides to decline any treatment and let nature take care of her. All attempts to persuade her fail. You document these discussions and arrange for follow-up visits. One month later, she is brought to the emergency department by her family members due to blurry vision, headache, confusion and mucosal bleeding. A detailed history from the family members fail to identify any precipitating factor. They all agree that, "She was in her usual state of health this morning, when she suddenly started to experience the symptoms." What is the most likely cause of this patient's condition? B. C. D. E. A. Disseminated intravascular coagulation Hyperviscosity syndrome Hypercalcemia Acute renal failure Acute liver failure

This subscription is licensed to user: roopika only User ID: 123489 Explanation: Hyperviscosity syndrome results from increased serum viscosity due to high protein content. It can be observed in patients with multiple myeloma, although it is more commonly seen in Waldenstrom' s macroglobulinemia as a result of increased amounts of IgM. Serum hyperviscosity impairs microcirculation in the brain, thereby causing headache, dizziness, vertigo, nystagmus, hearing loss and visual impairment. In severe cases, somnolence, coma and seizures may result. Mucosal hemorrhage is also typical due to impaired platelet function. (Choices C and D) Two other diagnoses that should be considered in multiple myeloma patients with neurological symptoms include acute renal failure and hypercalcemia. Acute renal failure can cause both neurological symptoms and bleeding tendency, but it is

usually less sudden in onset; furthermore, other prominent symptoms which are absent in this patient (and therefore exclude this diagnosis) are edema, nausea, vomiting, and decreased urination. Hypercalcemia can also cause neurological symptoms, but it is not associated with mucosal bleeding or blurry vision. (Choices A and E) Disseminated intravascular coagulation and acute liver failure are unlikely in this patient. Educational Objective: Hyperviscosity syndrome results from increased serum viscosity due to high protein content. It can be observed in patients with multiple myeloma, although it is more commonly seen in Waldenstrom' s macroglobulinemia. *Extremely important question for USMLE step-3 54% of people answered this question correctly; This subscription is licensed to user ID: 123489 only A 72-year-old Caucasian male comes to your office. He is complaining of hoarseness for the past six months. He says the hoarseness has been gradually worsening, and now he has developed difficulty in swallowing, pain in both ears, and exertional shortness of breath. He adds that he recently had a couple of episodes of hemoptysis. On examination, significant cervical lymphadenopathy is present. Chest x-ray is normal. Flexible fiberoptic endoscopy shows a laryngeal mass involving both supra glottic and infra glottic regions, as well as the vocal cords. The biopsy report reveals squamous cell carcinoma. The surgery and otolaryngology team, after careful review of all the findings, declare the lesion inoperable. At this time, what is the most appropriate treatment option for your patient? B. C. D. A. Radiotherapy alone Chemotherapy alone Comfort measures only Combined radiotherapy and chemotherapy

This subscription is licensed to user: roopika only User ID: 123489 Explanation: Over 50,000 new cases of head and neck cancer (HNC) are diagnosed every year in the United States. At the time of diagnosis, about 60% of these cases are regionally advanced (stage lll and lV). Wide surgical excision is not the immediate therapeutic option for most of these locally advanced HNC. A number of studies have shown that a course of combined chemotherapy and radiotherapy, also known as chemoradiotherapy (CRT), promises superior results over chemotherapy or radiotherapy alone. It not only increases the 5-year survival rate, but also renders some inoperable HNC cases operable after treatment.

(Choice A) Radiotherapy alone results in a 5-year survival rate of 10-30 percent. However, this rate still remains lower than that of CRT. (Choice B) Chemotherapy alone has produced a good response in a number of patients. This mode of management may render some laryngeal masses operable from their previously inoperable state. However, combined chemotherapy and radiotherapy promises superior results over chemotherapy alone. (Choice C) With CRT, high survival rates have been demonstrated. Unless the patient refuses to consent to chemotherapy or radiotherapy, just doing comfort measures without trying to treat the patient is inappropriate. Educational Objective: About 60% of head and neck cancers are locally advanced at the time of diagnosis and are inoperable. CRT promises superior results than chemotherapy or radiotherapy alone. 52% of people answered this question correctly; This subscription is licensed to user ID: 123489 only A 58-year-old Caucasian man comes to the office with complaints of cough for the past two months and blood-tinged sputum for the past three days. He has lost approximately 25 pounds over the past six months. He denies any history of low-grade fevers or night sweats. He has a long history of heavy smoking and chronic obstructive pulmonary disease. His temperature is 37.0C (99F), blood pressure is 120/62 mmHg, and heart rate is 82/min. He is a thin, cachectic man who is in no acute distress. Lung auscultation reveals the presence of rales in the right upper lung fields. His white cell count is 8,000/cubic mm with normal differential count. His serum calcium level is 12.8 mg/dL. His chest x-rays shows a 3.2 cm mass in the right upper lung zone, which is suspicious for a lung malignancy. Sputum cytology reveals poorly differentiated squamous cell carcinoma. Pulmonary function testing shows that his FEV1 is 1200 cc, while quantitative ventilation-perfusion scan shows that 65% of his pulmonary function comes from his right lung. What is the next best step in the management of this patient? B. C. D. E. A. Right pneumonectomy CT scan of the chest Right upper lobectomy Radiation and chemotherapy Mediastinoscopy

This subscription is licensed to user: roopika only User ID: 123489 Explanation: Lung cancer is the most common cause of cancer death in the United States and throughout the world. The appropriate selection of therapy, as well as the patient's prognosis, depends on the histologic type and accurate staging of the tumor. The important components of the current TNM staging system include the extent of local

tumor spread, the presence or absence of nodal involvement, and distant metastases. The presence of local or distant nodal spread or distant metastases to other body parts greatly influences the treatment approaches in patients with lung cancer. It is therefore important to accurately stage the patient before initiating any specific treatment. This is done through a detailed clinical assessment, radiographic imaging via CT scan or PET scan, and radionuclide bone scans. An initial CT scan should be obtained in all patients with lung cancer (small cell or nonsmall cell lung cancer) to aid in the initial staging work-up. It is extremely useful in detecting any mediastinal lymph node metastases and chest wall invasion. It also allows for accurate measurement of the tumor size, detection of small pleural effusions, and evaluation of liver and adrenal glands for metastatic disease. In easily accessible lesions, it can be useful in obtaining a CT-guided biopsy for tissue diagnosis of lung cancer. For these reasons, this patient should have a CT scan before planning or initiating any further treatment. (Choices A and C) Surgical resection is the treatment of choice for patients with stage I and some stage II non-small cell lung cancers; however, it is not as effective in patients with stage III disease, and most patients require adjuvant radiation and chemotherapy as well. The exact stage of disease in this patient is unknown, and he should not be referred for lobectomy or pneumonectomy at this point. Moreover, the patient in the vignette has a very low preoperative FEV1, and the quantitative ventilation-perfusion scanning shows that a significant portion of his pulmonary reserve comes from his right lung. He would therefore be unable to tolerate a right pneumonectomy. (Choice D) Adjuvant radiation and chemotherapy has been shown to improve the outcomes in patients who undergo local tumor resection; however, radiation or chemotherapy should not be planned, unless a definite and accurate staging of the tumor has been done. Baseline CT scan is required to document the tumor response to chemo/radiation. (Choice E) Mediastinoscopy with mediastinal lymph node sampling is indicated to document the presence or absence of malignancy in patients with suspected nodal involvement on chest CT scanning. Educational Objective: Accurate histological identification and staging of the tumor, along with preoperative physiologic assessment of lung function, is an important initial step in the optimal management of patients with lung cancer. 50% of people answered this question correctly; This subscription is licensed to user ID: 123489 only A 65-year-old Caucasian female presents to the emergency department with progressive shortness of breath on minimal exertion and fatigue. She was diagnosed with

myelodysplasia two years ago, and has been receiving supportive therapy with frequent RBC transfusions. Her last transfusion was 3 months ago. She experienced an episode of severe pneumonia one year ago that required hospitalization and IV antibiotic therapy. She is taking no medications currently and has no known allergies. Her blood pressure is 120/70 mmHg and heart rate is 95/min. Physical examination reveals pallor. Systolic murmur with intensity of II/VI is heard over the cardiac apex. Laboratory findings are significant for hematocrit of 24% and hemoglobin level of 5.7 mg/dL. You consider RBC transfusion in this patient. Blood grouping and cross-matching are done, but the blood bank is unable to find suitable blood. This is the first time such an incompatibility has occurred. What is the most likely reason for this incompatibility? B. C. D. E. A. Autoantibodies Alloantibodies Anti-Rh (D) antibodies Anti-HLA antibodies ABO incompatibility

This subscription is licensed to user: roopika only User ID: 123489 Explanation: After blood is ordered for transfusion, the following compatibility testing is usually performed. First, the patient's ABO and Rh types are determined. After this, the patient's serum is screened for unexpected antibodies, a procedure called pretransfusion antibody screening. Pretransfusion antibody screening is intended to detect any of all clinically significant RBC antibodies. If negative, the patient can be safely transfused. If positive, further investigation is usually warranted to evaluate the identity of the antibody. The major problem that leads to difficulties finding cross-matched blood in patients with a history of multiple transfusions is alloantibodies (e.g., in patients with sickle cell anemia or myelodysplasia). The most commonly implicated RBC antigens in that case are E, L and K. Moreover, these patients tend to develop multiple alloantibodies that make finding compatible blood even more difficult. (Choices C and E) Finding ABO and Rh-compatible blood is usually not a big challenge. (Choice A) Autoantibodies are less likely to cause difficulties in cross-matching in this patient; they are commonly implicated as a cause of the incompatible cross-match in patients with autoimmune anemia and taking certain drugs (e.g., methyldopa and procainamide). (Choice D) HLA allosensitization increases risk of graft rejection in patients awaiting organ or bone marrow transplantation and platelet refractoriness in those requiring subsequent platelet transfusion support. Remember that RBCs do not express HLA antigens. Educational Objective:

The major problem that leads to difficulties finding cross-matched blood in patients with a history of multiple transfusions is alloantibodies. 51% of people answered this question correctly; This subscription is licensed to user ID: 123489 only A 55-year-old Caucasian woman comes to see you in the office for a follow-up visit. She has a history of breast cancer in the left breast two years ago, for which she underwent a mastectomy. Her breast cancer was found to be estrogen receptor positive, and she was started on tamoxifen for adjuvant hormone therapy at that time. The patient has recently learned on television that tamoxifen can cause an increase in the incidence of uterine cancer. She wants you to "check her completely" for any evidence of uterine cancer. Which of the following is the most appropriate surveillance strategy for uterine cancer in patients on tamoxifen? B. C. D. A. Careful history and annual Pap smears Semiannual transvaginal ultrasound for endometrial thickening Annual CT scan of the pelvis for endometrial hyperplasia Annual endometrial biopsy

This subscription is licensed to user: roopika only User ID: 123489 Explanation: Tamoxifen is used as an adjuvant endocrine therapy for hormone responsive (estrogen receptor positive) breast cancer. It is also approved for the prevention of breast cancer in women with a high risk of breast cancer; however, its use has been associated with the development of endometrial hyperplasia and cancer. Most of these cancers are localized, stage I lesions, and are seen in women over 50 years of age. There are no clear evidence-based guidelines for routine gynecologic surveillance of patients on tamoxifen. The American College of Obstetrics and Gynecology recommends performing an annual gynecologic examination, including a detailed history and annual Pap smears, on all patients on tamoxifen. (Choice B) A transvaginal ultrasound can be used to evaluate endometrial thickening and hyperplasia; however, it is associated with significant false positive results which require subsequent endometrial sampling. (Choice C) Annual CT scans of the pelvis for the evaluation of the endometrial lining is not recommended for uterine cancer screening. (Choice D) A routine annual endometrial biopsy is not recommended in asymptomatic patients on tamoxifen. Educational Objective:

An annual gynecologic examination with a complete history and Pap smear is recommended to screen for endometrial cancer in patients on tamoxifen therapy. 31% of people answered this question correctly; This subscription is licensed to user ID: 123489 only The following vignette applies to the next 2 items A 52-year-old Caucasian woman comes to the office for a routine follow-up visit. She complains of weakness and fatigue for the past two weeks. She 'feels tired' even while doing minor household chores. She denies fever or any changes in appetite or weight. Three months ago, she underwent surgery for carcinoma of the breast and has been on chemotherapy ever since. She is a retired nurse and lives at home with her husband. She was diagnosed with hypertension twenty years ago. Her current therapy includes cyclophosphamide, methotrexate, 5-flurouracil, ondansetron, atenolol and famotidine. Her vital signs are within normal limits. Examination is unremarkable, except for moderate pallor. Rectal examination shows no abnormalities. Her lab results reveal the following: CBCHb 8.5g/dL MCV 112 fl Platelet count 100,000/cmm Leukocyte count 3,500/cmm Segmented neutrophils 68% Lymphocytes 26% Monocytes 6% Serum chemistrySerum Na 135 mEq/L Serum K 3.4 mEq/L Chloride 106 mEq/L Bicarbonate 20 mEq/L BUN 16 mg/dL Serum creatinine 1.0 mg/dL Calcium 9.0 mg/dL Blood Glucose 118 mg/dL Item 1 of 2 The stool test for occult blood is negative. Which of the following is most likely responsible for her condition? B. C. A. Cyclophosphamide Methotrexate 5- Fluorouracil

D. E.

Ondansetron Famotidine

This subscription is licensed to user: roopika only User ID: 123489 Explanation: This patient's laboratory results reveal a macrocytic anemia. Of all the drugs that she is currently taking, methotrexate is the drug that most commonly results in a macrocytic anemia. This side effect is attributed to the fact that methotrexate inhibits the enzyme dihydrofolate reductase (DHFR), which is essential for the conversion of folic acid to its reduced form (FH4- folinic acid), which can be utilized by the cells. The resulting folate deficiency then leads to the macrocytic anemia. Educational Objective: Trimethoprim, methotrexate, and phenytoin can interfere with folate metabolism and result in folic acid deficiency anemia. 76% of people answered this question correctly; This subscription is licensed to user ID: 123489 only Item 2 of 2 Which of the following is the most appropriate pharmacotherapy for this patient? B. C. D. E. A. Vitamin B12 Oral ferrous sulphate Parenteral ferrous sulphate Folic acid Folinic acid

This subscription is licensed to user: roopika only User ID: 123489 Explanation: Folinic acid is more potent than folic acid in 'rescuing' red blood cells from the deficiency by bypassing the block on DHFR. It is therefore the drug of choice for folate deficiency anemia induced by chronic, high dose methotrexate therapy. (Choice D) Folic acid may be used as a supplement to prevent or treat the folate deficiency seen with chronic use of methotrexate. It is not the drug of choice. (Choice A) Vitamin B12 therapy is inappropriate since this patient is most likely deficient in folic acid. (Choices B and D) Ferrous sulphate treatment is also inappropriate since the patient does not have iron deficiency.

Educational Objective: Chronic methotrexate therapy results in folate depletion and produces a macrocytic anemia. It is best treated with folinic acid (Leucovorin). 30% of people answered this question correctly; This subscription is licensed to user ID: 123489 only A 63-year-old Caucasian woman is hospitalized because of headaches, weakness, weight loss, heartburn, abdominal pain, abdominal distention, diarrhea, and black stools. She has had these symptoms for the past few weeks. Her other medical problems include hypertension, hypercholesterolemia, peripheral vascular disease, and bronchial asthma. Her father had prostate cancer, and her mother had colon cancer. Her medications include lisinopril, albuterol and ipratropium metered dose inhalers. She is allergic to cats. Her temperature is 37.2 C (99 F), blood pressure is 130/80 mmHg, pulse is 104/min, and respirations are 16/min. Examination shows no cardiovascular or respiratory abnormalities. The abdomen is soft, mildly tender and non-distended; bowel sounds are increased. There is no rebound tenderness or rigidity. There is no costovertebral angle tenderness. Neurologic examination is within normal limits. Fecal occult blood per rectum is positive. The patient's labs reveal: CBCHb 9 g/dL Ht 28% MCV 76fl Platelet count 600,000/cmm Leukocyte count 13,000/cmm Segmented neutrophils 76% Eosinophils 9% Lymphocytes 14% Monocytes 1% Serum ChemistrySerum Na 131 mEq/L Serum K 4.6 mEq/L Chloride 90 mEq/L Bicarbonate 16 mEq/L BUN 48 mg/dL Serum Creatinine 1.7 mg/dL Calcium 11.4 mg/dL Her chest-x ray shows no infiltrates, but there are some sclerotic lesions in the right humerus. Which of the following is the most likely diagnosis? B. C. A. Mastocytosis Whipple's disease Scleroderma

D. E.

Primary hyperparathyroidism Multiple myeloma

This subscription is licensed to user: roopika only User ID: 123489 Explanation: The presence of sclerotic bone lesions, diarrhea, eosinophilia, and peptic ulcer disease or gastrointestinal bleeding, are essential features of systemic mastocytosis. (Choice C) Scleroderma usually doesn't cause bone damage. (Choice D) Hyperparathyroidism would not explain the eosinophilia or diarrhea. (Choice B) Whipple's disease is not associated with sclerotic bone lesions. (Choice E) Multiple myeloma bone lesions are typically osteolytic, not sclerotic. Educational Objective: Systemic mastocytosis should be suspected in any patient with unexplained peptic disease, malabsorption, eosinophilia, unexplained flushing or anaphylaxis, and sclerotic bone lesions. H1 receptor antagonists, proton pump inhibitors, or cromolyn sodium are part of the initial management of this disease. 19% of people answered this question correctly; This subscription is licensed to user ID: 123489 only

Anda mungkin juga menyukai